Как найти область значений квадратичной функции: алгебра / Найти область значений квадратичной функции / Математика

Содержание

Квадратичная функция, парабола, график, свойства: нули, вершина, ось симметрии, промежутки возрастания, убывания. Тесты

Тестирование онлайн

  • Квадратичная функция

Определение. График

Квадратичной (квадратной) функцией называется функция вида

где a, b, с — числа.

Графиком квадратичной функции является парабола.

Парабола имеет вершину, ось, проведенная через вершину и параллельная оси Оу, делит параболу на две симметричные части. Вершиной параболы называется точка

Если коэффициент а>0, то ветви параболы направлены вверх, если a, то ветви параболы направлены вниз.

Свойства квадратичной функции y=x

2

1) Областью определения функции является множество всех действительных чисел, т.е.

2) Множеством значений функции является промежуток

3) Значение функции y=0 является наименьшим, а наибольшего значения функция не имеет.

4) Функция является четной, график симметричен относительно оси Оу.

5) Функция непериодическая.

6)Парабола имеет с осями координат единственную общую точку (0;0) — начало координат.

7) Значение аргумента x=0 является нулем функции.

8) На промежутке функция убывающая, а на промежутке — возрастающая.

9) Функция принимает положительные значения на множестве , т.е. все точки параболы, кроме начала координат.

Преобразование параболы

Функция y=x2 — частный случай квадратичной функции.

Квадратичную функцию всегда можно привести у виду , а затем построить параболу с помощью ее геометрических преобразований.

Для построения параболы необходимо:

1) Найти координаты вершины

2) Построить ось симметрии, проанализировать куда направлены ветви параболы

3) Найти точки пересечения параболы с осью Ox (нули), если они есть, решив уравнение

4) Найти точку пересечения с осью Оу, решив уравнение

Область значений функции

Всем здравствуйте! Тренируемся находить область значений функции! Кто еще не понял, что такое область определения (а она нам тоже понадобится непременно), тому сюда.

Что же такое область значений функции? Это та “часть” оси ординат, та область, где можно наткнуться на какие-либо точки, принадлежащие функции. То есть можно сказать, что если область значений найдена, то все точки функции находятся в ней, не выше и не ниже. Это почти тоже самое, что и область определения, только теперь это “область определения по оси ординат”. Здесь никаких особых ограничений нет, поэтому, чтобы найти область значений, нужно иметь представление об элементарных функциях – например, как выглядят парабола или гипербола, как определить, направлены ли ветви параболы вверх или вниз и т.п. Все это рассказано и показано здесь.

Ну, поехали!

Примеры.

1. Найдите область значений функции

Решение: функция – квадратичная, представляет собой параболу с положительным старшим коэффициентом, ветви направлены вверх. Понятно тогда, что весь график располагается выше координаты своей вершины (вершина – самая низшая точка). Ордината вершины: , тогда .

 

2. Найдите область значений функции

Решение: область определения функции ( ] [).

В точках (-7) и (3) двучлен обращается в ноль. Поскольку результат извлечения корня – величина положительная, то вся функция располагается выше оси абсцисс, и ее область значений [)

 

3. Найти область значений функции

Область определения – вся числовая ось, кроме ноля. Можем подставить любое число из области определения, при этом функция всегда отрицательна.

Из графика также видно, что

4. Найти область значений функции:

Решение. Область определения:

На концах отрезка функция принимает значение 1, под корнем имеем квадратный двучлен, наибольшее значение он принимает в вершине, при , значит, функция будет принимать в этой точке наименьшее значение.

Подставив 1, получаем

Ответ:

5. Найдите область значений функции:

Очевидно, что график данной функции может быть получен из графика обычной параболы , область значений которой легко найти: ветви направлены вверх, поэтому низшая точка – вершина параболы. Однако заметим также, что если аргумент функции под знаком модуля, то график такой функции может быть построен с помощью отражения части  графика, лежащей в правой вертикальной полуплоскости, в левую полуплоскость(см. рисунок). Тогда от нашей параболы останется только часть, лежащая правее оси ординат, и именно она будет отражена относительно оси y, и тогда низшей точкой окажется та, в которой график пересечет ось ординат, а это – значение свободного члена (коэффициента с), который у нас равен (-6).

Область значений нашей функции [)

6. Найдите область значений функции:

Очевидно, что график данной функции может быть получен из графика обычной параболы . Так как все выражение находится под знаком модуля, то для  того, чтобы построить такой график, нужно отразить всю часть графика, расположенную ниже оси х, вверх, поэтому [).

 

 

 

 

 

 

 

7. Найдите область значений функции:

Данная функция получена преобразованием обычной гиперболы. Данная функция не существует при , или .  При  второе слагаемое обращается в ноль, и функция стремится к  значению , причем можно заметить, что при положительных больших значениях х данная функция приближается к 2 снизу, а при отрицательных  – сверху.

Ответ:

 

8. Найти область значений функции:

Решение. Область определения:

При  функция принимает наибольшее значение ,

При x, стремящемся к бесконечности, функция стремится к нулю. Но мы запишем область значений от меньшего к большему:

Ответ: (]

Как определить область значения параболы

Тестирование онлайн

Определение. График

Квадратичной (квадратной) функцией называется функция вида

где a, b, с — числа.

Графиком квадратичной функции является парабола.

Парабола имеет вершину, ось, проведенная через вершину и параллельная оси Оу, делит параболу на две симметричные части. Вершиной параболы называется точка

Если коэффициент а>0, то ветви параболы направлены вверх, если a 2

1) Областью определения функции является множество всех действительных чисел, т.е.

2) Множеством значений функции является промежуток

3) Значение функции y=0 является наименьшим, а наибольшего значения функция не имеет.

4) Функция является четной, график симметричен относительно оси Оу.

6)Парабола имеет с осями координат единственную общую точку (0;0) — начало координат.

7) Значение аргумента x=0 является нулем функции.

8) На промежутке функция убывающая, а на промежутке — возрастающая.

9) Функция принимает положительные значения на множестве , т.е. все точки параболы, кроме начала координат.

Преобразование параболы

Функция y=x 2 — частный случай квадратичной функции.

Квадратичную функцию всегда можно привести у виду , а затем построить параболу с помощью ее геометрических преобразований.

Для построения параболы необходимо:

1) Найти координаты вершины

2) Построить ось симметрии, проанализировать куда направлены ветви параболы

3) Найти точки пересечения параболы с осью Ox (нули), если они есть, решив уравнение

4) Найти точку пересечения с осью Оу, решив уравнение

Зачастую в рамках решения задач нам приходится искать множество значений функции на области определения или отрезке. Например, это нужно делать при решении разных типов неравенств, оценках выражений и др.

В рамках этого материала мы расскажем, что из себя представляет область значений функции, приведем основные методы, которыми ее можно вычислить, и разберем задачи различной степени сложности. Для наглядности отдельные положения проиллюстрированы графиками. Прочитав эту статью, вы получите исчерпывающее представление об области значений функции.

Начнем с базовых определений.

Множество значений функции y = f ( x ) на некотором интервале x представляет собой множество всех значений, которые данная функция принимает при переборе всех значений x ∈ X .

Область значений функции y = f ( x ) – это множество всех ее значений, которые она может принять при переборе значений x из области x ∈ ( f ) .

Область значений некоторой функции принято обозначать E ( f ) .

Обратите внимание, что понятие множества значений функции не всегда тождественно области ее значений. Эти понятия будут равнозначны только в том случае, если интервал значений x при нахождении множества значений совпадет с областью определения функции.

Важно также различать область значений и область допустимых значений переменной x для выражения в правой части y = f ( x ) . Область допустимых значений x для выражения f ( x ) и будет областью определения данной функции.

Ниже приводится иллюстрация, на которой показаны некоторые примеры. Синие линии – это графики функций, красные – асимптоты, рыжие точки и линии на оси ординат – это области значений функции.

Очевидно, что область значений функции можно получить при проецировании графика функции на ось O y . При этом она может представлять собой как одно число, так и множество чисел, отрезок, интервал, открытый луч, объединение числовых промежутков и др.

Рассмотрим основные способы нахождения области значений функции.

Начнем с определения множества значений непрерывной функции y = f ( x ) на некотором отрезке, обозначенном [ a ; b ] . Мы знаем, что функция, непрерывная на некотором отрезке, достигает на нем своего минимума и максимума, то есть наибольшего m a x x ∈ a ; b f ( x ) и наименьшего значения m i n x ∈ a ; b f ( x ) . Значит, у нас получится отрезок m i n x ∈ a ; b f ( x ) ; m a x x ∈ a ; b f ( x ) , в котором и будут находиться множества значений исходной функции. Тогда все, что нам нужно сделать, – это найти на этом отрезке указанные точки минимума и максимума.

Возьмем задачу, в которой нужно определить область значений арксинуса.

Условие: найдите область значений y = a r c sin x .

Решение

В общем случае область определения арксинуса располагается на отрезке [ — 1 ; 1 ] . Нам надо определить наибольшее и наименьшее значение указанной функции на нем.

y ‘ = a r c sin x ‘ = 1 1 — x 2

Мы знаем, что производная функции будет положительной для всех значений x , расположенных в интервале [ — 1 ; 1 ] , то есть на протяжении всей области определения функция арксинуса будет возрастать. Значит, самое маленькое значение она примет при x , равном — 1 , а самое большое – при x , равном 1 .

m i n x ∈ — 1 ; 1 a r c sin x = a r c sin — 1 = — π 2 m a x x ∈ — 1 ; 1 a r c sin x = a r c sin 1 = π 2

Таким образом, область значений функции арксинус будет равна E ( a r c sin x ) = — π 2 ; π 2 .

Ответ: E ( a r c sin x ) = — π 2 ; π 2

Условие: вычислите область значений y = x 4 — 5 x 3 + 6 x 2 на заданном отрезке [ 1 ; 4 ] .

Решение

Все, что нам нужно сделать, – это вычислить наибольшее и наименьшее значение функции в заданном интервале.

Для определения точек экстремума надо произвести следующие вычисления:

y ‘ = x 4 — 5 x 3 + 6 x 2 ‘ = 4 x 3 + 15 x 2 + 12 x = x 4 x 2 — 15 x + 12 y ‘ = 0 ⇔ x ( 4 x 2 — 15 x + 12 ) = 0 x 1 = 0 ∉ 1 ; 4 и л и 4 x 2 — 15 x + 12 = 0 D = — 15 2 — 4 · 4 · 12 = 33 x 2 = 15 — 33 8 ≈ 1 . 16 ∈ 1 ; 4 ; x 3 = 15 + 33 8 ≈ 2 . 59 ∈ 1 ; 4

Теперь найдем значения заданной функции в концах отрезка и точках x 2 = 15 — 33 8 ; x 3 = 15 + 33 8 :

y ( 1 ) = 1 4 — 5 · 1 3 + 6 · 1 2 = 2 y 15 — 33 8 = 15 — 33 8 4 — 5 · 15 — 33 8 3 + 6 · 15 — 33 8 2 = = 117 + 165 33 512 ≈ 2 . 08 y 15 + 33 8 = 15 + 33 8 4 — 5 · 15 + 33 8 3 + 6 · 15 + 33 8 2 = = 117 — 165 33 512 ≈ — 1 . 62 y ( 4 ) = 4 4 — 5 · 4 3 + 6 · 4 2 = 32

Значит, множество значений функции будет определяться отрезком 117 — 165 33 512 ; 32 .

Ответ: 117 — 165 33 512 ; 32 .

Перейдем к нахождению множества значений непрерывной функции y = f ( x ) в промежутках ( a ; b ) , причем a ; + ∞ , — ∞ ; b , — ∞ ; + ∞ .

Начнем с определения наибольшей и наименьшей точки, а также промежутков возрастания и убывания на заданном интервале. После этого нам нужно будет вычислить односторонние пределы в концах интервала и/или пределы на бесконечности. Иными словами, нам надо определить поведении функции в заданных условиях. Для этого у нас есть все необходимые данные.

Условие: вычислите область значений функции y = 1 x 2 — 4 на интервале ( — 2 ; 2 ) .

Решение

Определяем наибольшее и наименьшее значение функции на заданном отрезке

y ‘ = 1 x 2 — 4 ‘ = — 2 x ( x 2 — 4 ) 2 y ‘ = 0 ⇔ — 2 x ( x 2 — 4 ) 2 = 0 ⇔ x = 0 ∈ ( — 2 ; 2 )

У нас получилось максимальное значение, равное 0 , поскольку именно в этой точке происходит перемена знака функции и график переходит к убыванию. См. на иллюстрацию:

То есть y ( 0 ) = 1 0 2 — 4 = — 1 4 будет максимальным значений функции.

Теперь определим поведение функции при таком x, который стремится к — 2 с правой стороны и к + 2 с левой стороны. Иными словами, найдем односторонние пределы:

lim x → — 2 + 0 1 x 2 — 4 = lim x → — 2 + 0 1 ( x — 2 ) ( x + 2 ) = = 1 — 2 + 0 — 2 — 2 + 0 + 2 = — 1 4 · 1 + 0 = — ∞ lim x → 2 + 0 1 x 2 — 4 = lim x → 2 + 0 1 ( x — 2 ) ( x + 2 ) = = 1 2 — 0 — 2 2 — 0 + 2 = 1 4 · 1 — 0 = — ∞

У нас получилось, что значения функции будут возрастать от минус бесконечности до — 1 4 тогда, когда аргумент изменяется в пределах от — 2 до 0 . А когда аргумент меняется от 0 до 2 , значения функции убывают к минус бесконечности. Следовательно, множеством значений заданной функции на нужном нам интервале будет ( — ∞ ; — 1 4 ] .

Ответ: ( — ∞ ; — 1 4 ] .

Условие: укажите множество значений y = t g x на заданном интервале — π 2 ; π 2 .

Решение

Нам известно, что в общем случае производная тангенса в — π 2 ; π 2 будет положительной, то есть функция будет возрастать. Теперь определим, как ведет себя функция в заданных границах:

lim x → π 2 + 0 t g x = t g — π 2 + 0 = — ∞ lim x → π 2 — 0 t g x = t g π 2 — 0 = + ∞

Мы получили рост значений функции от минус бесконечности к плюс бесконечности при изменении аргумента от — π 2 до π 2 ,и можно сказать, что множеством решений данной функции будет множество всех действительных чисел.

Ответ: — ∞ ; + ∞ .

Условие: определите, какова область значений функции натурального логарифма y = ln x .

Решение

Нам известно, что данная функция является определенной при положительных значениях аргумента D ( y ) = 0 ; + ∞ . Производная на заданном интервале будет положительной: y ‘ = ln x ‘ = 1 x . Значит, на нем происходит возрастание функции. Далее нам нужно определить односторонний предел для того случая, когда аргумент стремится к 0 (в правой части), и когда x стремится к бесконечности:

lim x → 0 + 0 ln x = ln ( 0 + 0 ) = — ∞ lim x → ∞ ln x = ln + ∞ = + ∞

Мы получили, что значения функции будут возрастать от минус бесконечности до плюс бесконечности при изменении значений x от нуля до плюс бесконечности. Значит, множество всех действительных чисел – это и есть область значений функции натурального логарифма.

Ответ: множество всех действительных чисел – область значений функции натурального логарифма.

Условие: определите, какова область значений функции y = 9 x 2 + 1 .

Решение

Данная функция является определенной при условии, что x – действительное число. Вычислим наибольшие и наименьшие значения функции, а также промежутки ее возрастания и убывания:

y ‘ = 9 x 2 + 1 ‘ = — 18 x ( x 2 + 1 ) 2 y ‘ = 0 ⇔ x = 0 y ‘ ≤ 0 ⇔ x ≥ 0 y ‘ ≥ 0 ⇔ x ≤ 0

В итоге мы определили, что данная функция будет убывать, если x ≥ 0 ; возрастать, если x ≤ 0 ; она имеет точку максимума y ( 0 ) = 9 0 2 + 1 = 9 при переменной, равной 0 .

Посмотрим, как же ведет себя функция на бесконечности:

lim x → — ∞ 9 x 2 + 1 = 9 — ∞ 2 + 1 = 9 · 1 + ∞ = + 0 lim x → + ∞ 9 x 2 + 1 = 9 + ∞ 2 + 1 = 9 · 1 + ∞ = + 0

Из записи видно, что значения функции в этом случае будут асимптотически приближаться к 0.

Подведем итоги: когда аргумент изменяется от минус бесконечности до нуля, то значения функции возрастают от 0 до 9 . Когда значения аргумента меняются от 0 до плюс бесконечности, соответствующие значения функции будут убывать от 9 до 0 . Мы отобразили это на рисунке:

На нем видно, что областью значений функции будет интервал E ( y ) = ( 0 ; 9 ]

Ответ: E ( y ) = ( 0 ; 9 ]

Если нам надо определить множество значений функции y = f ( x ) на промежутках [ a ; b ) , ( a ; b ] , [ a ; + ∞ ) , ( — ∞ ; b ] , то нам понадобится провести точно такие же исследования. Эти случаи мы пока не будем разбирать: далее они нам еще встретятся в задачах.

А как быть в случае, если область определения некоторой функции представляет из себя объединение нескольких промежутков? Тогда нам надо вычислить множества значений на каждом из этих промежутков и объединить их.

Условие: определите, какова будет область значений y = x x — 2 .

Решение

Поскольку знаменатель функции не должен быть обращен в 0 , то D ( y ) = — ∞ ; 2 ∪ 2 ; + ∞ .

Начнем с определения множества значений функции на первом отрезке — ∞ ; 2 , который представляет из себя открытый луч. Мы знаем, что функция на нем будет убывать, то есть производная данной функции будет отрицательной.

lim x → 2 — 0 x x — 2 = 2 — 0 2 — 0 — 2 = 2 — 0 = — ∞ lim x → — ∞ x x — 2 = lim x → — ∞ x — 2 + 2 x — 2 = lim x → — ∞ 1 + 2 x — 2 = 1 + 2 — ∞ — 2 = 1 — 0

Тогда в тех случаях, когда аргумент изменяется по направлению к минус бесконечности, значения функции будут асимптотически приближаться к 1 . Если же значения x меняются от минус бесконечности до 2 , то значения будут убывать от 1 до минус бесконечности, т.е. функция на этом отрезке примет значения из интервала — ∞ ; 1 . Единицу мы исключаем из наших рассуждений, поскольку значения функции ее не достигают, а лишь асимптотически приближаются к ней.

Для открытого луча 2 ; + ∞ производим точно такие же действия. Функция на нем также является убывающей:

lim x → 2 + 0 x x — 2 = 2 + 0 2 + 0 — 2 = 2 + 0 = + ∞ lim x → + ∞ x x — 2 = lim x → + ∞ x — 2 + 2 x — 2 = lim x → + ∞ 1 + 2 x — 2 = 1 + 2 + ∞ — 2 = 1 + 0

Значения функции на данном отрезке определяются множеством 1 ; + ∞ . Значит, нужная нам область значений функции, заданной в условии, будет объединением множеств — ∞ ; 1 и 1 ; + ∞ .

Ответ: E ( y ) = — ∞ ; 1 ∪ 1 ; + ∞ .

Это можно увидеть на графике:

Особый случай – периодические функции. Их область значения совпадает с множеством значений на том промежутке, который отвечает периоду этой функции.

Условие: определите область значений синуса y = sin x .

Решение

Синус относится к периодической функции, а его период составляет 2 пи. Берем отрезок 0 ; 2 π и смотрим, каким будет множество значений на нем.

y ‘ = ( sin x ) ‘ = cos x y ‘ = 0 ⇔ cos x = 0 ⇔ x = π 2 + πk , k ∈ Z

В рамках 0 ; 2 π у функции будут точки экстремума π 2 и x = 3 π 2 . Подсчитаем, чему будут равны значения функции в них, а также на границах отрезка, после чего выберем самое большое и самое маленькое значение.

y ( 0 ) = sin 0 = 0 y π 2 = sin π 2 = 1 y 3 π 2 = sin 3 π 2 = — 1 y ( 2 π ) = sin ( 2 π ) = 0 ⇔ min x ∈ 0 ; 2 π sin x = sin 3 π 2 = — 1 , max x ∈ 0 ; 2 π sin x = sin π 2 = 1

Ответ: E ( sin x ) = — 1 ; 1 .

Если вам нужно знать области значений таких функций, как степенная, показательная, логарифмическая, тригонометрическая, обратная тригонометрическая, то советуем вам перечитать статью об основных элементарных функциях. Теория, которую мы приводим здесь, позволяет проверить указанные там значения. Их желательно выучить, поскольку они часто требуются при решении задач. Если вы знаете области значений основных функций, то легко сможете находить области функций, которые получены из элементарных с помощью геометрического преобразования.

Условие: определите область значения y = 3 a r c cos x 3 + 5 π 7 — 4 .

Решение

Нам известно, что отрезок от 0 до пи есть область значений арккосинуса. Иными словами, E ( a r c cos x ) = 0 ; π или 0 ≤ a r c cos x ≤ π . Мы можем получить функцию a r c cos x 3 + 5 π 7 из арккосинуса, сдвинув и растянув ее вдоль оси O x , но такие преобразования нам ничего не дадут. Значит, 0 ≤ a r c cos x 3 + 5 π 7 ≤ π .

Функция 3 a r c cos x 3 + 5 π 7 может быть получена из арккосинуса a r c cos x 3 + 5 π 7 с помощью растяжения вдоль оси ординат, т.е. 0 ≤ 3 a r c cos x 3 + 5 π 7 ≤ 3 π . Финалом преобразований является сдвиг вдоль оси O y на 4 значения. В итоге получаем двойное неравенство:

0 — 4 ≤ 3 a r c cos x 3 + 5 π 7 — 4 ≤ 3 π — 4 ⇔ — 4 ≤ 3 arccos x 3 + 5 π 7 — 4 ≤ 3 π — 4

Мы получили, что нужная нам область значений будет равна E ( y ) = — 4 ; 3 π — 4 .

Ответ: E ( y ) = — 4 ; 3 π — 4 .

Еще один пример запишем без пояснений, т.к. он полностью аналогичен предыдущему.

Условие: вычислите, какова будет область значений функции y = 2 2 x — 1 + 3 .

Решение

Перепишем функцию, заданную в условии, как y = 2 · ( 2 x — 1 ) — 1 2 + 3 . Для степенной функции y = x — 1 2 область значений будет определена на промежутке 0 ; + ∞ , т.е. x — 1 2 > 0 . В таком случае:

2 x — 1 — 1 2 > 0 ⇒ 2 · ( 2 x — 1 ) — 1 2 > 0 ⇒ 2 · ( 2 x — 1 ) — 1 2 + 3 > 3

Значит, E ( y ) = 3 ; + ∞ .

Ответ: E ( y ) = 3 ; + ∞ .

Теперь разберем, как найти область значений функции, которая не является непрерывной. Для этого нам надо разбить всю область на промежутки и найти множества значений на каждом из них, после чего объединить то, что получилось. Чтобы лучше понять это, советуем повторить основные виды точек разрыва функции.

Условие: дана функция y = 2 sin x 2 — 4 , x ≤ — 3 — 1 , — 3 x ≤ 3 1 x — 3 , x > 3 . Вычислите область ее значений.

Решение

Данная функция является определенной для всех значений x . Проведем ее анализ на непрерывность при значениях аргумента, равных — 3 и 3 :

lim x → — 3 — 0 f ( x ) = lim x → — 3 2 sin x 2 — 4 = 2 sin — 3 2 — 4 = — 2 sin 3 2 — 4 lim x → — 3 + 0 f ( x ) = lim x → — 3 ( 1 ) = — 1 ⇒ lim x → — 3 — 0 f ( x ) ≠ lim x → — 3 + 0 f ( x )

Имеем неустранимый разрыв первого рода при значении аргумента — 3 . При приближении к нему значения функции стремятся к — 2 sin 3 2 — 4 , а при стремлении x к — 3 с правой стороны значения будут стремиться к — 1 .

lim x → 3 — 0 f ( x ) = lim x → 3 — 0 ( — 1 ) = 1 lim x → 3 + 0 f ( x ) = lim x → 3 + 0 1 x — 3 = + ∞

Имеем неустранимый разрыв второго рода в точке 3 . Когда функция стремится к нему, ее значения приближаются к — 1 , при стремлении к той же точке справа – к минус бесконечности.

Значит, вся область определения данной функции является разбитой на 3 интервала ( — ∞ ; — 3 ] , ( — 3 ; 3 ] , ( 3 ; + ∞ ) .

На первом из них у нас получилась функция y = 2 sin x 2 — 4 . Поскольку — 1 ≤ sin x ≤ 1 , получаем:

— 1 ≤ sin x 2 1 ⇒ — 2 ≤ 2 sin x 2 ≤ 2 ⇒ — 6 ≤ 2 sin x 2 — 4 ≤ — 2

Значит, на данном промежутке ( — ∞ ; — 3 ] множество значении функции – [ — 6 ; 2 ] .

На полуинтервале ( — 3 ; 3 ] получилась постоянная функция y = — 1 . Следовательно, все множество ее значений в данном случае будет сводится к одному числу — 1 .

На втором промежутке 3 ; + ∞ у нас есть функция y = 1 x — 3 . Она является убывающей, потому что y ‘ = — 1 ( x — 3 ) 2 0 . Она будет убывать от плюс бесконечности до 0 , но самого 0 не достигнет, потому что:

lim x → 3 + 0 1 x — 3 = 1 3 + 0 — 3 = 1 + 0 = + ∞ lim x → + ∞ 1 x — 3 = 1 + ∞ — 3 = 1 + ∞ + 0

Значит, множество значений исходной функции при x > 3 представляет собой множество 0 ; + ∞ . Теперь объединим полученные результаты: E ( y ) = — 6 ; — 2 ∪ — 1 ∪ 0 ; + ∞ .

Ответ: E ( y ) = — 6 ; — 2 ∪ — 1 ∪ 0 ; + ∞ .

Решение показано на графике:

Условие: есть функция y = x 2 — 3 e x . Определите множество ее значений.

Решение

Она определена для всех значений аргумента, представляющих собой действительные числа. Определим, в каких промежутках данная функция будет возрастать, а в каких убывать:

y ‘ = x 2 — 3 e x ‘ = 2 x e x — e x ( x 2 — 3 ) e 2 x = — x 2 + 2 x + 3 e x = — ( x + 1 ) ( x — 3 ) e x

Мы знаем, что производная обратится в 0 , если x = — 1 и x = 3 . Поместим эти две точки на ось и выясним, какие знаки будет иметь производная на получившихся интервалах.

Функция будет убывать на ( — ∞ ; — 1 ] ∪ [ 3 ; + ∞ ) и возрастать на [ — 1 ; 3 ] . Точкой минимума будет — 1 , максимума – 3 .

Теперь найдем соответствующие значения функции:

y ( — 1 ) = — 1 2 — 3 e — 1 = — 2 e y ( 3 ) = 3 2 — 3 e 3 = 6 e — 3

Посмотрим на поведение функции на бесконечности:

lim x → — ∞ x 2 — 3 e x = — ∞ 2 — 3 e — ∞ = + ∞ + 0 = + ∞ lim x → + ∞ x 2 — 3 e x = + ∞ 2 — 3 e + ∞ = » open=» + ∞ + ∞ = = lim x → + ∞ x 2 — 3 ‘ e x ‘ = lim x → + ∞ 2 x e x = » open=» + ∞ + ∞ = = lim x → + ∞ 2 x ‘ ( e x ) ‘ = 2 lim x → + ∞ 1 e x = 2 · 1 + ∞ = + 0

Для вычисления второго предела было использовано правило Лопиталя. Изобразим ход нашего решения на графике.

На нем видно, что значения функции будут убывать от плюс бесконечности до — 2 e тогда, когда аргумент меняется от минус бесконечности до — 1 . Если же он изменяется от 3 до плюс бесконечности, то значения будут убывать от 6 e — 3 до 0 , но при этом 0 достигнут не будет.

Таким образом, E ( y ) = [ — 2 e ; + ∞ ) .

Ответ: E ( y ) = [ — 2 e ; + ∞ )

  • 5 — 9 классы
  • Алгебра
  • 8 баллов

как найти область значений функции у параболы

  • Попроси больше объяснений
  • Следить
  • Отметить нарушение

Zonder555 18.04.2013

Ответ

Проверено экспертом

1) Если функция f(x) задана графиком, то нужно найти на графике координату у₀ вершины параболы.

Если ветви параболы направлены вниз, то область значений E(f)=(-∞;y₀].

Если ветви параболы направлены вверх, то область значений E(f)=[y₀;+∞).

2) Если функция задана формулой f(x) = ax²+bx+c, то сначала нужно найти координаты вершины. Сначала координату х : . Затем ее подставить в формулу и найти координату у₀=f(x₀)=ax₀²+bx₀+c.

Если a>0, то область значений функции E(f)=[y₀;+∞).

Если а 0, то область значений функции E(f)=[y₀;+∞).

«>

Как найти область определения функции

После этого экскурса в важную составную матанализа многие согласятся, что найти
область определения функции не очень сложно. Ненамного сложнее, чем Московскую область на карте.

Во-первых, нужно различать виды функций (корень, дробь, синус и др.). Во-вторых,
решать уравнения и неравенства с учетом вида функции (например, на что нельзя делить, какое выражение
не может быть под знаком корня и тому подобное). Согласитесь, не так уж много и не так сложно.

Итак, чтобы находить области определения распространённых функций, порешаем
уравнения и неравенства с одной переменной. А в конце урока обобщим понятие на уровне теории. Пока же —
краткое определение. Область определения функции y=f(x)
— это множество значений X, для которых существуют значения Y
.

Будут и задачи для самостоятельного решения, к которым можно
посмотреть ответы.

Приступаем к практике. На рисунке изображён график функции .
Знаменатель дроби не может быть равен нулю, так как на нуль делить нельзя. Поэтому, приравнивая знаменатель
нулю, получаем значение, не входящее в область определения функции: 1. То есть, область определения заданной функции —
это все значения «икса» от минус бесконечности до единицы и от единицы до плюс бесконечности
. Это хорошо
видно на графике. Приведённый здесь пример функции относится к виду дробей. На уроке разберём решения
всех распространённых видов функций.

Пример 0. Как найти область
определения функции игрек равен квадратному корню из икса минус пять (подкоренное выражение икс минус пять)
()? Нужно всего лишь
решить неравенство

x — 5 ≥ 0,

так как для того, чтобы мы получили действительное значение игрека, подкоренное
выражение должно быть больше или равно нулю. Получаем решение: область определения функции — все значения икса
больше или равно пяти (или икс принадлежит промежутку от пяти включительно до плюс бесконечности).

На чертеже сверху — фрагмент числовой оси. На ней область опредения рассмотренной функции
заштрихована, при этом в «плюсовом» направлении штриховка продолжается бесконечно вместе с самой осью.

Постоянная (константа) определена при любых действительных
значениях x, следовательно, данная функция определена на всём
множестве R действительных чисел. Это можно записать и так:
областью определения данной функции является вся числовая прямая ]- ∞; + ∞[.

Пример 1. Найти область определения функции
y = 2.

Решение. Область определения функции не указана, значит, в силу выше приведённого
определения имеется в виду естественная область определения. Выражение
f(x) = 2 определено при любых действительных
значениях x, следовательно, данная функция определена на всём
множестве R действительных чисел.

Поэтому на чертеже сверху числовая прямая заштрихована на всём протяжении от минус
бесконечности до плюс бесконечности.

В случае, когда функция задана формулой и n — натуральное число:

Пример 2. Найти область определения функции
.

Решение. Как следует из
определения, корень чётной степени имеет смысл, если подкоренное выражение неотрицательно, то есть,
если — 1 ≤ x ≤ 1.
Следовательно, область определения данной функции — [- 1; 1].

Заштрихованная область числовой прямой на чертеже сверху — это область определения
данной функции.

Область определения степенной функции с целым показателем степени

В случае, когда функция задана формулой :

если a — положительное, то областью определения функции является множество
всех действительных чисел, то есть ]- ∞; + ∞[;

если a — отрицательное, то областью определения функции является
множество ]- ∞; 0[ ∪ ]0 ;+ ∞[,
то есть вся числовая прямая за исключением нуля.

На соответствующем чертеже сверху вся числовая прямая заштрихована, а точка,
соответствующая нулю, выколота (она не входит в область определения функции).

Пример 3. Найти область определения функции
.

Решение. Первое слагаемое целой степенью икса, равной 3, а степень икса во втором
слагаемом можно представить в виде единицы — так же целого числа.
Следовательно, область определения данной функции — вся числовая прямая, то есть
]- ∞; + ∞[.

Область определения степенной функции с дробным показателем степени

В случае, когда функция задана формулой :

если
— положительное, то областью определения функции является множество [0; + ∞[;

если
— отрицательное, то областью определения функции является множество ]0; + ∞[.

Пример 4. Найти область определения функции
.

Решение. Оба слагаемых в выражении функции — степенные функции с положительными
дробными показателями степеней. Следовательно, область определения данной функции —
множество [0; + ∞[.

На чертеже сверху заштрихована часть числовой прямой от нуля (включительно) и больше,
причём штриховка продолжается вместе с самой прямой до плюс бесконечности.

Пример 5. Найти область определения функции
.

Решение. Дробный показатель степени данной степенной функции — отрицательный.
Поэтому решим строгое неравенство, когда квадратный трёхчлен в скобках строго больше нуля::

.

Дикриминант получился отрицательный. Следовательно сопряжённое неравенству
квадратное уравнение не имеет корней. А это значит, что квадратный трёхчлен ни при каких значениях
«икса» не равен нулю. Таким образом, область определения данной функции — вся числовая ось, или,
что то же самое — множество R действительных чисел, или,
что то же самое — ]- ∞; + ∞[.

Область определения показательной функции

В случае, когда функция задана формулой ,
областью определения функции является вся числовая прямая, то есть
]- ∞; + ∞[.

Область определения логарифмической функции

Логарифмическая функция
определена при условии, если её аргумент положителен, то есть, областью её определения является множество
]0; + ∞[.

Найти область определения функции самостоятельно, а затем посмотреть решение


Область определения функции y = cos(x) —
так же множество R действительных чисел.

Область определения функции y = tg(x) —
множество R действительных чисел, кроме чисел
.

Область определения функции y = ctg(x) —
множество R действительных чисел, кроме чисел
.

Пример 8. Найти область определения функции
.

Решение. Внешняя функция — десятичный логарифм и на область её определения
распространяются условия области определения логарифмической функции вообще. То есть, её аргумент
должен быть положительным. Аргумент здесь — синус «икса». Поворачивая воображаемый циркуль по
окружности, видим, что условие sin x > 0
нарушается при «иксе» равным нулю, «пи», два, умноженном на «пи» и вообще равным произведению числа «пи»
и любого чётного или нечётного целого числа.

Таким образом, область определения данной функции задаётся выражением

,

где k — целое число.

Область определения обратных тригонометрических функций

Область определения функции y = arcsin(x) —
множество [-1; 1].

Область определения функции y = arccos(x) —
так же множество [-1; 1].

Область определения функции y = arctg(x) —
множество R действительных чисел.

Область определения функции y = arcctg(x) —
так же множество R действительных чисел.

Пример 9. Найти область определения функции
.

Решение. Решим неравенство:

Таким образом, получаем область определения данной функции — отрезок
[- 4; 4].

Пример 10. Найти область определения функции
.

Решение. Решим два неравенства:

Решение первого неравенства:

Решение второго неравенства:

Таким образом, получаем область определения данной функции — отрезок
[0; 1].

Если функция задана дробным выражением, в котором переменная находится в знаменателе
дроби, то областью определения функции является множество R действительных чисел,
кроме таких x, при которых знаменатель дроби обращается в нуль.

Пример 11. Найти область определения функции
.

Решение. Решая равенство нулю знаменателя дроби, находим область определения данной функции — множество
]- ∞; — 2[ ∪ ]- 2 ;+ ∞[.

Пример 12. Найти область определения функции
.

Решение. Решим уравнение:

Таким образом, получаем область определения данной функции —
]- ∞; — 1[ ∪ ]- 1 ; 1[ ∪ ]1 ;+ ∞[.

Пример 13. Найти область определения функции
.

Решение. Область определения первого слагаемого — данной функции — множество
R действительных чисел, второго слагаемого — все
действительные числа, кроме -2 и 2 (получили, решая равенство нулю знаменателя, как в предыдущем примере). В этом случае область определения функции должна удовлетворять
условиями определения обоих слагаемых. Следовательно, область определения данной функции — все
x, кроме -2 и 2.

Пример 14. Найти область определения функции
.

Решение. Решим уравнение:

Уравнение не имеет действительных корней. Но функция определена только на действительных
числах. Таким образом, получаем область определения данной функции — вся числовая прямая или, что
то же самое — множество R действительных чисел или,
что то же самое — ]- ∞; + ∞[.

То есть, какое бы число мы не подставляли вместо «икса», знаменатель никогда не
будет равен нулю.

Пример 15. Найти область определения функции
.

Решение. Решим уравнение:

Таким образом, получаем область определения данной функции —
]- ∞; — 1[ ∪ ]- 1 ; 0[ ∪ ]0 ; 1[ ∪ ]1 ;+ ∞[.

Пример 16. Найти область определения функции
.

Решение. Кроме того, что знаменатель не может быть равным нулю, ещё и выражение под
корнем не может быть отрицательным. Сначала решим уравнение:

График квадратичной функции под корнем представляет собой параболу, ветви которой
направлены вверх. Как следует из решения квадратного уравнения, парабола пересекает ось Ox в точках
1 и 2. Между этими точками линия параболы находится ниже оси Ox, следовательно значения
квадратичной функции между этими точками отрицательное. Таким образом, исходная функция не определена
на отрезке [1; 2].

Найти область определения функции самостоятельно, а затем посмотреть решение


Если функция задана формулой вида y = kx + b,
то область определения функции — множество
R действительных чисел.

А теперь обобщим решения рассмотренных примеров. Каждой точке графика функции соответствуют:

  • определённое значение «икса» — аргумента функции;
  • определённое значение «игрека» — самой функции.

Верны следующие факты.

  • От аргумента — «икса» — вычисляется «игрек» — значения функции.
  • Область определения функции — это множества всех значений «икса», для которых существует, то есть может
    быть вычислен «игрек» — значение функции. Иначе говоря, множество значений аргумента, на котором
    «функция работает».

Весь раздел «Исследование функций»

Область значений функции (множество значений функции). Необходимые понятия и примеры нахождения

Зачастую в рамках решения задач нам приходится искать множество значений функции на области определения или отрезке. Например, это нужно делать при решении разных типов неравенств, оценках выражений и др.

В рамках этого материала мы расскажем, что из себя представляет область значений функции, приведем основные методы, которыми ее можно вычислить, и разберем задачи различной степени сложности. Для наглядности отдельные положения проиллюстрированы графиками. Прочитав эту статью, вы получите исчерпывающее представление об области значений функции.

Начнем с базовых определений.

Определение 1

Множество значений функции y = f(x) на некотором интервале x представляет собой множество всех значений, которые данная функция принимает при переборе всех значений x∈X.

Определение 2

Область значений функции y=f(x) – это множество всех ее значений, которые она может принять при переборе значений x из области x∈(f).

Область значений некоторой функции принято обозначать E(f).

Обратите внимание, что понятие множества значений функции не всегда тождественно области ее значений. Эти понятия будут равнозначны только в том случае, если интервал значений x при нахождении множества значений совпадет с областью определения функции.

 Важно также различать область значений и область допустимых значений переменной x для выражения в правой части y=f(x). Область допустимых значений x для выражения f(x) и будет областью определения данной функции.

Ниже приводится иллюстрация, на которой показаны некоторые примеры. Синие линии – это графики функций, красные – асимптоты, рыжие точки и линии на оси ординат – это области значений функции.

Очевидно, что область значений функции можно получить при проецировании графика функции на ось Oy. При этом она может представлять собой как одно число, так и множество чисел, отрезок, интервал, открытый луч, объединение числовых промежутков и др.

Рассмотрим основные способы нахождения области значений функции.

Начнем с определения множества значений непрерывной функции y = f(x) на некотором отрезке, обозначенном [a; b]. Мы знаем, что функция, непрерывная на некотором отрезке, достигает на нем своего минимума и максимума, то есть наибольшего maxx∈a; bf(x) и наименьшего значения minx∈a; bf(x). Значит, у нас получится отрезок minx∈a; bf(x); maxx∈a; bf(x), в котором и будут находиться множества значений исходной функции. Тогда все, что нам нужно сделать, – это найти на этом отрезке указанные точки минимума и максимума.

Возьмем задачу, в которой нужно определить область значений арксинуса.

Пример 1

Условие: найдите область значений y = arcsin x.

Решение

В общем случае область определения арксинуса располагается на отрезке [-1; 1]. Нам надо определить наибольшее и наименьшее значение указанной функции на нем.

y’ = arcsin x’=11-x2

Мы знаем, что производная функции будет положительной для всех значений x, расположенных в интервале [-1; 1], то есть на протяжении всей области определения функция арксинуса будет возрастать. Значит, самое маленькое значение она примет при x, равном -1, а самое большое – при x, равном 1.

minx∈-1; 1arcsin x=arcsin-1=-π2maxx∈-1; 1arcsin x=arcsin 1=π2

Таким образом, область значений функции арксинус будет равна E(arcsin x)=-π2; π2.

Ответ:  E(arcsin x)=-π2; π2

Пример 2

Условие: вычислите область значений y=x4-5×3+6×2 на заданном отрезке [1; 4].

Решение 

Все, что нам нужно сделать, – это вычислить наибольшее и наименьшее значение функции в заданном интервале.

Для определения точек экстремума надо произвести следующие вычисления:

y’=x4-5×3+6×2’=4×3+15×2+12x=x4x2-15x+12y’=0⇔x(4×2-15x+12)=0x1=0∉1; 4 или 4×2-15x+12=0D=-152-4·4·12=33×2=15-338≈1. 16∈1; 4; x3=15+338≈2.59∈1; 4

Теперь найдем значения заданной функции в концах отрезка и точках x2=15-338; x3=15+338:

y(1)=14-5·13+6·12=2y15-338=15-3384-5·15-3383+6·15-3382==117+16533512≈2.08y15+338=15+3384-5·15+3383+6·15+3382==117-16533512≈-1.62y(4)=44-5·43+6·42=32

Значит, множество значений функции будет определяться отрезком 117-16533512; 32.

Ответ: 117-16533512; 32.

Перейдем к нахождению множества значений непрерывной функции y = f(x) в промежутках (a; b), причем a; +∞, -∞; b, -∞; +∞.

Начнем с определения наибольшей и наименьшей точки, а также промежутков возрастания и убывания на заданном интервале. После этого нам нужно будет вычислить односторонние пределы в концах интервала и/или пределы на бесконечности. Иными словами, нам надо определить поведении функции в заданных условиях. Для этого у нас есть все необходимые данные.

Пример 3

Условие: вычислите область значений функции y=1×2-4 на интервале (-2; 2).

Решение

Определяем наибольшее и наименьшее значение функции на заданном отрезке

y’=1×2-4’=-2x(x2-4)2y’=0⇔-2x(x2-4)2=0⇔x=0∈(-2; 2)

У нас получилось максимальное значение, равное 0, поскольку именно в этой точке происходит перемена знака функции и график переходит к убыванию. См. на иллюстрацию:

То есть  y(0)=102-4=-14 будет максимальным значений функции.

Теперь определим поведение функции при таком x, который стремится к -2 с правой стороны и к +2 с левой стороны. Иными словами, найдем односторонние пределы:

limx→-2+01×2-4=limx→-2+01(x-2)(x+2)==1-2+0-2-2+0+2=-14·1+0=-∞limx→2+01×2-4=limx→2+01(x-2)(x+2)==12-0-22-0+2=14·1-0=-∞

У нас получилось, что значения функции будут возрастать от минус бесконечности до -14 тогда, когда аргумент изменяется в пределах от -2 до 0. А когда аргумент меняется от 0 до 2, значения функции убывают к минус бесконечности. Следовательно, множеством значений заданной функции на нужном нам интервале будет (-∞; -14].

Ответ: (-∞; -14].

Пример 4

Условие: укажите множество значений y=tg x на заданном интервале -π2; π2.

Решение

Нам известно, что в общем случае производная тангенса в -π2; π2 будет положительной, то есть функция будет возрастать. Теперь определим, как ведет себя функция в заданных границах:

limx→π2+0tg x=tg-π2+0=-∞limx→π2-0tg x=tgπ2-0=+∞

Мы получили рост значений функции от минус бесконечности к плюс бесконечности при изменении аргумента от -π2 до π2,и можно сказать, что множеством решений данной функции будет множество всех действительных чисел.

Ответ: -∞; +∞.

Пример 5

Условие: определите, какова область значений функции натурального логарифма y = ln x.

Решение

Нам известно, что данная функция является определенной при положительных значениях аргумента D(y)=0; +∞. Производная на заданном интервале будет положительной: y’=ln x’=1x. Значит, на нем происходит возрастание функции. Далее нам нужно определить односторонний предел для того случая, когда аргумент стремится к 0 (в правой  части), и когда x стремится к бесконечности:

limx→0+0ln x=ln(0+0)=-∞limx→∞ln x=ln+∞=+∞

Мы получили, что значения функции будут возрастать от минус бесконечности до плюс бесконечности при изменении значений x от нуля до плюс бесконечности. Значит, множество всех действительных чисел – это и есть область значений функции натурального логарифма.

Ответ: множество всех действительных чисел – область значений функции натурального логарифма.

Нужна помощь преподавателя?

Опиши задание — и наши эксперты тебе помогут!

Описать задание

Пример 6

Условие: определите, какова область значений функции y=9×2+1.

Решение

Данная функция является определенной при условии, что x – действительное число. Вычислим наибольшие и наименьшие значения функции, а также промежутки ее возрастания и убывания:

y’=9×2+1’=-18x(x2+1)2y’=0⇔x=0y’≤0⇔x≥0y’≥0⇔x≤0

В итоге мы определили, что данная функция будет убывать, если x≥0; возрастать, если x≤0; она имеет точку максимума y(0)=902+1=9 при переменной, равной 0.

Посмотрим, как же ведет себя функция на бесконечности:

limx→-∞9×2+1=9-∞2+1=9·1+∞=+0limx→+∞9×2+1=9+∞2+1=9·1+∞=+0

Из записи видно, что значения функции в этом случае будут асимптотически приближаться к 0.

Подведем итоги: когда аргумент изменяется от минус бесконечности до нуля, то значения функции возрастают от 0 до 9. Когда значения аргумента меняются от 0 до плюс бесконечности, соответствующие значения функции будут убывать от 9 до 0. Мы отобразили это на рисунке:

На нем видно, что областью значений функции будет интервал E(y)=(0; 9]

Ответ: E(y)=(0; 9]

Если нам надо определить множество значений функции y = f(x) на промежутках [a; b), (a; b], [a; +∞), (-∞; b], то нам понадобится провести точно такие же исследования. Эти случаи мы пока не будем разбирать: далее они нам еще встретятся в задачах.

А как быть в случае, если область определения некоторой функции представляет из себя объединение нескольких промежутков? Тогда нам надо вычислить множества значений на каждом из этих промежутков и объединить их.

Пример 7

Условие: определите, какова будет область значений y=xx-2.

Решение

Поскольку знаменатель функции не должен быть обращен в 0, то D(y)=-∞; 2∪2; +∞.

Начнем с определения множества значений функции на первом отрезке -∞; 2, который представляет из себя открытый луч. Мы знаем, что функция на нем будет убывать, то есть производная данной функции будет отрицательной.

limx→2-0xx-2=2-02-0-2=2-0=-∞limx→-∞xx-2=limx→-∞x-2+2x-2=limx→-∞1+2x-2=1+2-∞-2=1-0

Тогда в тех случаях, когда аргумент изменяется по направлению к минус бесконечности, значения функции будут асимптотически приближаться к 1. Если же значения x меняются от минус бесконечности до 2, то значения будут убывать от 1 до минус бесконечности, т.е. функция на этом отрезке примет значения из интервала -∞; 1. Единицу мы исключаем из наших рассуждений, поскольку значения функции ее не достигают, а лишь асимптотически приближаются к ней.

Для открытого луча 2; +∞ производим точно такие же действия. Функция на нем также является убывающей:

limx→2+0xx-2=2+02+0-2=2+0=+∞limx→+∞xx-2=limx→+∞x-2+2x-2=limx→+∞1+2x-2=1+2+∞-2=1+0

Значения функции на данном отрезке определяются множеством 1; +∞. Значит, нужная нам область значений функции, заданной в условии, будет объединением множеств -∞; 1 и 1; +∞.

Ответ: E(y)=-∞; 1∪1; +∞.

Это можно увидеть на графике:

Особый случай – периодические функции. Их область значения совпадает с множеством значений на том промежутке, который отвечает периоду этой функции.

Пример 8

Условие: определите область значений синуса y = sin x.

Решение

Синус относится к периодической функции, а его период составляет 2 пи. Берем отрезок 0; 2π и смотрим, каким будет множество значений на нем.

y’=(sin x)’=cos xy’=0⇔cos x=0⇔x=π2+πk, k∈Z

В рамках 0; 2π у функции будут точки экстремума π2 и x=3π2. Подсчитаем, чему будут равны значения функции в них, а также на границах отрезка, после чего выберем самое большое и самое маленькое значение.

y(0)=sin 0=0yπ2=sin π2=1y3π2=sin3π2=-1y(2π)=sin(2π)=0⇔minx∈0; 2πsin x=sin3π2=-1, maxx∈0; 2πsin x=sinπ2=1

Ответ: E(sin x)=-1; 1.

Если вам нужно знать области значений таких функций, как степенная, показательная, логарифмическая, тригонометрическая, обратная тригонометрическая, то советуем вам перечитать статью об основных элементарных функциях. Теория, которую мы приводим здесь, позволяет проверить указанные там значения. Их желательно выучить, поскольку они часто требуются при решении задач. Если вы знаете области значений основных функций, то легко сможете находить области функций, которые получены из элементарных с помощью геометрического преобразования.

Пример 9

Условие: определите область значения y=3arccosx3+5π7-4.

Решение

Нам известно, что отрезок от 0 до пи есть область значений арккосинуса. Иными словами, E(arccos x)=0; π или 0≤arccos x≤π. Мы можем получить функцию arccosx3+5π7 из арккосинуса, сдвинув и растянув ее вдоль оси Ox, но такие преобразования нам ничего не дадут. Значит, 0≤arccosx3+5π7≤π.

Функция 3arccosx3+5π7 может быть получена из арккосинуса arccosx3+5π7 с помощью растяжения вдоль оси ординат, т.е. 0≤3arccosx3+5π7≤3π. Финалом преобразований является сдвиг вдоль оси Oy на 4 значения. В итоге получаем двойное неравенство:

0-4≤3arccosx3+5π7-4≤3π-4⇔-4≤3arccosx3+5π7-4≤3π-4

Мы получили, что нужная нам область значений будет равна E(y)=-4; 3π-4.

Ответ: E(y)=-4; 3π-4.

Еще один пример запишем без пояснений, т.к. он полностью аналогичен предыдущему.

Пример 10

Условие: вычислите, какова будет область значений функции y=22x-1+3.

Решение

Перепишем функцию, заданную в условии, как y=2·(2x-1)-12+3. Для степенной функции y=x-12 область значений будет определена на промежутке 0; +∞, т.е. x-12>0. В таком случае:

2x-1-12>0⇒2·(2x-1)-12>0⇒2·(2x-1)-12+3>3

Значит, E(y)=3; +∞.

Ответ: E(y)=3; +∞.

Теперь разберем, как найти область значений функции, которая не является непрерывной. Для этого нам надо разбить всю область на промежутки и найти множества значений на каждом из них, после чего объединить то, что получилось. Чтобы лучше понять это, советуем повторить основные виды точек разрыва функции.

Пример 11

Условие: дана функция y=2sinx2-4, x≤-3-1, -3<x≤31x-3, x>3. Вычислите область ее значений.

Решение

Данная функция является определенной для всех значений  x. Проведем ее анализ на непрерывность при значениях аргумента, равных -3 и 3:

limx→-3-0f(x)=limx→-32sinx2-4=2sin-32-4=-2sin32-4limx→-3+0f(x)=limx→-3(1)=-1⇒limx→-3-0f(x)≠limx→-3+0f(x)

Имеем неустранимый разрыв первого рода при значении аргумента -3. При приближении к нему значения функции стремятся к -2sin32-4, а при стремлении x к -3 с правой стороны значения будут стремиться к -1.

limx→3-0f(x)=limx→3-0(-1)=1limx→3+0f(x)=limx→3+01x-3=+∞

Имеем неустранимый разрыв второго рода в точке 3. Когда функция стремится к нему, ее значения приближаются к -1, при стремлении к той же точке справа – к минус бесконечности.

Значит, вся область определения данной функции является разбитой на 3 интервала (-∞; -3], (-3; 3], (3; +∞).

На первом из них у нас получилась функция y=2sinx2-4. Поскольку -1≤sin x≤1, получаем:

-1≤sinx2<1⇒-2≤2sinx2≤2⇒-6≤2sinx2-4≤-2

Значит, на данном промежутке (-∞; -3] множество значении функции – [-6;2].

На полуинтервале (-3; 3] получилась постоянная функция y =-1. Следовательно, все множество ее значений в данном случае будет сводится к одному числу -1.

На втором промежутке 3; +∞ у нас есть функция y=1x-3. Она является убывающей, потому что y’=-1(x-3)2<0. Она будет убывать от плюс бесконечности до 0, но самого 0 не достигнет, потому что:

limx→3+01x-3=13+0-3=1+0=+∞limx→+∞1x-3=1+∞-3=1+∞+0

Значит, множество значений исходной функции при x > 3 представляет собой множество 0; +∞. Теперь объединим полученные результаты: E(y)=-6; -2∪-1∪0; +∞.

Ответ: E(y)=-6; -2∪-1∪0; +∞.

Решение показано на графике:

Пример 12

Условие: есть функция y=x2-3ex. Определите множество ее значений.

Решение

Она определена для всех значений аргумента, представляющих собой действительные числа. Определим, в каких промежутках данная функция будет возрастать, а в каких убывать:

y’=x2-3ex’=2xex-ex(x2-3)e2x=-x2+2x+3ex=-(x+1)(x-3)ex

Мы знаем, что производная обратится в 0, если x=-1 и x=3. Поместим эти две точки на ось и выясним, какие знаки будет иметь производная на получившихся интервалах.

Функция будет убывать на (-∞; -1]∪[3; +∞) и возрастать на [-1; 3]. Точкой минимума будет -1, максимума –3.

Теперь найдем соответствующие значения функции:

y(-1)=-12-3e-1=-2ey(3)=32-3e3=6e-3

Посмотрим на поведение функции на бесконечности:

limx→-∞x2-3ex=-∞2-3e-∞=+∞+0=+∞limx→+∞x2-3ex=+∞2-3e+∞=+∞+∞==limx→+∞x2-3’ex’=limx→+∞2xex=+∞+∞==limx→+∞2x'(ex)’=2limx→+∞1ex=2·1+∞=+0

Для вычисления второго предела было использовано правило Лопиталя. Изобразим ход нашего решения на графике.

На нем видно, что значения функции будут убывать от плюс бесконечности до -2e тогда, когда аргумент меняется от минус бесконечности до -1. Если же он изменяется от 3 до плюс бесконечности, то значения будут убывать от 6e-3 до 0, но при этом 0 достигнут не будет.

Таким образом, E(y)=[-2e; +∞).

Ответ:  E(y)=[-2e; +∞)

Функции и Графики — сайт по математике и не только!!! Всё о Математических функциях и их графиках…



КВАДРАТИЧНАЯ ФУНКЦИЯ


y = ax2 + bx + c, где a 0.

График квадратичной функции — парабола.


Свойства функции и вид её графика определяются, в основном, значениями коэффициента a
и дискриминанта
D = b2 — 4ac.

a > 0, D > 0a > 0, D = 0a > 0, D
a D > 0a D = 0a D


РАЗЛИЧНЫЕ ПРЕДСТАВЛЕНИЯ КВАДРАТИЧНОЙ ФУНКЦИИ

  1. ВЫДЕЛЕНИЕ ПОЛНОГО КВАДРАТА:
  2. РАЗЛОЖЕНИЕ НА ЛИНЕЙНЫЕ МНОЖИТЕЛИ

    при D > 0 y = ax2 + bx + c = a(x — x1)(x — x2)

    при D = 0 y = ax2 + bx + c = a(x — x1)2
    при D


СВОЙСТВА КВАДРАТИЧНОЙ ФУНКЦИИ
НАПРАВЛЕНИЕ ВЕТВЕЙ, ХАРАКТЕРНЫЕ ТОЧКИ И ОСЬ СИММЕТРИИ ПАРАБОЛЫ,

являющейся графиком функции у = ax2 + bx + c

  • Направление ветвей параболы:


    при a > 0 ветви направлены вверх


    при a
  • Координаты вершины параболы: (-b/2a; —D/4a)
  • Ось симметрии параболы — прямая
  • Точки пересечения (касания) графика с осью х:


    D > 0: (точки пересечения)
    D = 0: x1 = — b/(2a) (точка касания)
    D х нет


  • Точка пересечения графика с осью у:(0,c),
    симметричная ей точка относительно параболы (-b/a;c)


Для построения графика квадратичной функции можно использовать некоторые из указанных характеристик.
Например, если уравнение ax2 + bx + c = 0 имеет два корня, удобно использовать координаты вершины параболы и координаты двух точек пересечения параболы с осью х.




ПОСТРОЕНИЕ ГРАФИКА КВАДРАТИЧНОЙ ФУНКЦИИ ПО НАПРАВЛЕНИЮ ВЕТВЕЙ, ХАРАКТЕРНЫМ ТОЧКАМ И ОСИ СИММЕТРИИ ПАРАБОЛЫ

Примеры:


ПОСТРОЕНИЕ ГРАФИКА КВАДРАТИЧНОЙ ФУНКЦИИ С ПОМОЩЬЮ ЭЛЕМЕНТАРНЫХ ПРЕОБРАЗОВАНИЙ ГРАФИКА ФУНКЦИИ y = x2

С помощью выделения полного квадрата любую квадратичную функцию можно представить в виде:


Это свойство позволяет построить график квадратичной функции с помощью элементарных преобразований графика функции y = x2.


Построение графика y = a(x — m)2 + n можно произвести в три этапа:



Результат преобразования: график функции y = a(x — m)2+n

Примеры:

1. Растяжение графика функции y = x 2 вдоль оси y в 2 раза2. Параллельный перенос графика функции y = 2x 2 вдоль оси x на 3 вправоПараллельный перенос графика функции
y = 2(x — 3)2 вдоль оси y на 1 вверх.
1. Сжатие графика функции y = x 2 вдоль оси y в 2 раза и преобразование симметрии относительно оси x2. Параллельный перенос графика функции y = — x 2 вдоль оси x на 2 влевоПараллельный перенос графика функции
y = — (x + 2)2/ 2


Используются технологии uCoz

Квадратичная функция — PDF Free Download

Степенная функция. Функция вида y=x k, где k>0 постоянная, называется степенной функцией. Если k=1, то y=x линейная функция, ее график прямая линия.

Степенная функция Функция вида y=x k, где k>0 постоянная, называется степенной функцией. Если k=1, то y=x линейная функция, ее график прямая линия. Если k=2, то y=x 2 квадратичная функция, ее график парабола.

Подробнее

Исследование квадратного трехчлена

Исследование квадратного тречлена Пусть f(x) = ax 2 + bx + c имеет действительные корни x 1 и x 2, а M какое-нибудь действительное число, D = b 2 4ас — дискриминант При решении конкретны задач нужно особо

Подробнее

Пусть задано числовое множество D

Пусть задано числовое множество D R. Если каждому числу x D поставлено в соответствие единственное число y, то говорят, что на множестве D задана числовая функция: y = f (x), x D. Множество D, называется

Подробнее

Домашняя работа по алгебре за 9 класс

ВЕ Бачурин Домашняя работа по алгебре за 9 класс к учебнику «Алгебра: Учеб для 9 кл общеобразоват учреждений / ЮН Макарычев, НГ Миндюк, КИ Нешков, СБ Суворова; Под ред СА Теляковского 0-е изд М: Просвещение,

Подробнее

Урок на тему: Построение графиков.

Урок на тему: Построение графиков. Ребята, мы с вами строили уже не мало графиков функций, например параболы, гиперболы, тригонометрических функций и другие. Давайте вспомним, как мы это делали? Мы выбирали

Подробнее

Тема 41 «Задания с параметром»

Тема 41 «Задания с параметром» Основные формулировки заданий с параметром: 1) Найти все значения параметра, при каждом из которых выполняется определенное условие. ) Решить уравнение или неравенство с

Подробнее

Определение 1. Функция y = ax + bx + c, где a, b, c — действительные числа, причем a 0, называется квадратичной. 1) Область определения. ( f ) R.

СВОЙСТВА ФУНКЦИИ И ЕЕ ГРАФИК Определение. Функция, где,, — действительные числа, причем 0, называется квадратичной. Область определения. ( f R, так как выражение определено для любых. Область значений.

Подробнее

Критерии оценки заданий 18

Задание 18 Критерии оценки заданий 18 Содержание критерия Балл ы Обоснованно получен правильный ответ. 4 С помощью верного рассуждения получено множество значений а, отличающееся от искомого конечным числом

Подробнее

РАЗДЕЛ 14. ЗАДАЧИ С ПАРАМЕТРАМИ

РАЗДЕЛ ЗАДАЧИ С ПАРАМЕТРАМИ Комментарий Задачи с параметрами традиционно являются сложными заданиями в структуре ЕГЭ, требующими от абитуриента не только владения всеми методами и приемам решения различных

Подробнее

1) y=-x 2 +7x-14 2) y=x 2-7x+14 3) y=x 2 +7x+14 4) y=-x 2-7x-14

5.1: Установите соответствие между графиками функций и формулами, которые их задают. Впишите в приведённую в ответе таблицу под каждой буквой соответствующую цифру. ФОРМУЛЫ Графики 1) y=-x 2 +7x-14 2)

Подробнее

Тема 9 «Функция. Свойства функций»

Тема 9 «Функция. Свойства функций» Пусть X некоторое непустое множество действительных чисел. И пусть указан закон f, по которому каждому числу х ϵ X ставится в соответствие единственное число y ϵ Y, обозначаемое

Подробнее

Задание 18. Задачи с параметром

Линейное уравнение a x = b имеет: единственное решение, при a 0; бесконечное множество решений, при a = 0, b = 0; не имеет решений, при a = 0, b 0. Квадратное уравнение ax 2 + bx + c = 0 имеет: два различных

Подробнее

ГЛАВА II. Квадратный трехчлен

ГЛАВА II. Квадратный трехчлен Справочный материал Квадратным трехчленом называют выражение a + b + c, где abc,, и a 0. График квадратного трехчлена парабола. Прямая b = ее ось симметрии. Точка ( в; в)

Подробнее

Все прототипы задания В8 (2013)

Все прототипы задания В8 (13) ( 7485) Прямая y 7x 5 параллельна касательной к графику функции y x 6x 8 Найдите абсциссу точки касания ( 7486) Прямая y 4x 11 является касательной к графику функции 3 y x

Подробнее

Функция y = cos x. Ее свойства и график

Функция y = cos x Ее свойства и график 1 Сегодня мы рассмотрим Построение графика функции y = cos x; Свойства функции y = cos x; Изменение графика функции y = cos x в зависимости от изменения функции и

Подробнее

Элементы высшей математики

Кафедра математики и информатики Элементы высшей математики Учебно-методический комплекс для студентов СПО, обучающихся с применением дистанционных технологий Модуль Дифференциальное исчисление Составитель:

Подробнее

Построение графиков функций

Построение графиков функций 1. План исследования функции при построении графика 1. Найти область определения функции. Часто полезно учесть множество значений функции. Исследовать специальные свойства функции:

Подробнее

Дом Севастьянова. Екатеринбург

Дом Севастьянова. Екатеринбург 9. Функция =, её график и свойства 0. Свойства квадратных корней. Тождество =. Вынесение множителя из-под знака квадратного корня. Внесение множителя под знак квадратного

Подробнее

Глава 11 ТЕСТОВЫЕ ЗАДАНИЯ

Глава ТЕСТОВЫЕ ЗАДАНИЯ Т-0 Исследование функции по графику Т-0 Соответствие между графиком рациональной функции и формулой Т-0 Построение графика по свойствам Т-04 Параллельный перенос графика Т-05 Симметричное

Подробнее

Геометрический смысл производной

Геометрический смысл производной Рассмотрим график функции y=f(x) и касательную в точке P 0 (x 0 ; f(x 0 )). Найдем угловой коэффициент касательной к графику в этой точке. Угол наклона касательной Р 0

Подробнее

16.2.Н. Производная.

6..Н. Производная 6..Н. Производная. Оглавление 6..0.Н. Производная Введение…. 6..0.Н. Производная сложной функции…. 5 6..0.Н. Производные от функций с модулями…. 7 6..0.Н. Возрастание и убывание

Подробнее

Учебный центр «Резольвента»

ООО «Резольвента», www.resolventa.ru, [email protected], (495) 509-8-0 Учебный центр «Резольвента» Доктор физико-математических наук, профессор К. Л. САМАРОВ КВАДРАТНЫЙ ТРЕХЧЛЕН Учебно-методическое пособие

Подробнее

ТЕСТ Запишите координаты точек на координатной прямой, показанной на рисунке.

wwwaleeiivanovcom ДЗ Функции ТЕСТ 0 Запишите координаты точек на координатной прямой, показанной на рисунке ) G(-), C(-), K(-), A(4), J(0), M() ) G(-5), C(-6), K(-), A(9), J(0), M(5) ) G(-9), C(-5), K(-4),

Подробнее

Параметры и квадратный трёхчлен. 2

И. В. Яковлев Материалы по математике MathUs.ru Параметры и квадратный трёхчлен. 2 Данная статья посвящена вопросам расположения корней квадратного трёхчлена в зависимости от параметра. Вычисление корней

Подробнее

= 1 е) f(9) = 27; f(1) = 3

Глава 8 ФУНКЦИИ И ГРАФИКИ Алгоритмы А- Задание стандартных функций А- Понятие функции. График функции А-3 Каноническая запись зависимостей А- Задание стандартных функций. К стандартным функциям отнесем

Подробнее

8. Кривые второго порядка Окружность

8 Кривые второго порядка 81 Окружность Множество точек плоскости, равноудаленных от одной точки, называемой центром, на расстояние, называемое радиусом, называется окружностью Пусть центр окружности находится

Подробнее

и x 1x 2, в частности сумму одинаковых

Тема Квадратное уравнение Формулы Виета Два алгебраических выражения, соединенных знаком «=», образуют равенство Равенство, справедливое при всех допустимых значениях входящих в него переменных, называется

Подробнее

11.1. Функции Базовый уровень.

111 Функции Базовый уровень Оглавление 11101 Системы координат 1110 Понятие функции 7 1110 Область определения функции 10 11104 Область (множество) значений функции 1 11105 Возрастание и убывание функции

Подробнее

Математический анализ

Кафедра математики и информатики Математический анализ Учебно-методический комплекс для студентов ВПО, обучающихся с применением дистанционных технологий Модуль 4 Приложения производной Составитель: доцент

Подробнее

Тема 1.4 Функции, их свойства и графики

Тема.4 Функции, их свойства и графики Автор: Переверзьева Н.С. Преподаватель математики Лицей 6 Цели урока: Ознакомиться с понятием «функция», закрепить его на примерах Усвоить новые термины Узнать методы

Подробнее

Дифференциальное исчисление

Дифференциальное исчисление Основные понятия и формулы Определение 1 Производной функции в точке называется предел отношения приращения функции к приращению аргумента, при условии, что приращение аргумента

Подробнее

ID_4689 1/7 neznaika.pro

1 Производная и первообразная Ответами к заданиям являются слово, словосочетание, число или последовательность слов, чисел. Запишите ответ без пробелов, запятых и других дополнительных символов. На рисунке

Подробнее

ПРОМЕЖУТКИ ВОЗРАСТАНИЯ

1) На рисунке изображён график дифференцируемой функции y = f (x). На оси абсцисс отмечены девять точек:. Среди этих точек найдите все точки, в которых производная функции f (x) отрицательна. В ответе

Подробнее

Как найти диапазон парабол

Обновлено 19 декабря 2020 г.

Джек Джерард

В математике некоторые квадратичные функции создают так называемую параболу, когда вы их строите. Хотя ширина, расположение и направление параболы будут варьироваться в зависимости от конкретной функции, отображаемой на графике, все параболы обычно имеют U-образную форму (иногда с некоторыми дополнительными колебаниями в середине) и симметричны по обе стороны от их центральной точки ( также известный как вершина.) Если функция, которую вы графически представляете, является функцией с четным порядком, у вас будет парабола некоторого типа.

При работе с параболой есть несколько деталей, которые полезно вычислить. Одним из них является область параболы, которая указывает все возможные значения x , включенные в некоторую точку вдоль плеч параболы. Это довольно простой расчет, потому что рукава истинной параболы продолжают расширяться бесконечно; домен включает в себя все действительные числа. Другой полезный расчет — это диапазон параболы, который немного сложнее, но найти не так уж и сложно.

Область и диапазон графика

Область и диапазон параболы по существу относятся к тому, какие значения x и какие значения y включены в параболу (при условии, что парабола на стандартной двумерной оси x y .) Когда вы рисуете параболу на графике, может показаться странным, что область включает все действительные числа, потому что ваша парабола, скорее всего, выглядит как маленькая буква «U» на вашей оси.Однако в параболе есть нечто большее, чем вы видите; каждое плечо параболы должно заканчиваться стрелкой, указывающей на то, что она продолжается до ∞ (или до −∞, если ваша парабола обращена вниз). Это означает, что даже если вы ее не видите, парабола в конечном итоге расширится в обоих направлениях. направления достаточно большие, чтобы охватить все возможные значения x .

Однако то же самое не относится к оси y . Посмотрите на свою параболу еще раз. Даже если он расположен в самом низу вашего графика и открывается вверх, чтобы охватить все, что находится над ним, все равно есть более низкие значения y, которые вы просто не нарисовали на своем графике.На самом деле их бесконечное количество. Вы не можете сказать, что диапазон параболы включает в себя все действительные числа, потому что независимо от того, сколько чисел включает ваш диапазон, все равно существует бесконечное количество значений, которые выходят за пределы диапазона вашей параболы.

Параболы продолжаются вечно (в одном направлении)

Диапазон — это представление значений между двумя точками. Когда вы рассчитываете диапазон параболы, вы знаете только одну из этих точек для начала.Ваша парабола всегда будет идти вверх или вниз, поэтому конечное значение вашего диапазона всегда будет ∞ (или −∞, если ваша парабола направлена ​​вниз). Это полезно знать, потому что это означает, что половина работы определение диапазона уже сделано за вас еще до того, как вы начнете вычислять.

Если диапазон вашей параболы заканчивается на ∞, где он начинается? Посмотрите еще раз на свой график. Какое наименьшее значение y все еще включено в вашу параболу? Если парабола открывается вниз, ответьте на вопрос: какое наибольшее значение y входит в параболу? Каким бы ни было это значение, это начало вашей параболы.Если, например, самая низкая точка вашей параболы находится в начале координат — точка (0,0) на вашем графике — тогда самая низкая точка будет y = 0, а диапазон вашей параболы будет [0 , ∞) . При написании диапазона используйте квадратные скобки [] для чисел, включенных в диапазон (например, 0), и круглые скобки () для чисел, которые не включены (например, ∞, поскольку он никогда не может быть достигнут). n +.2 — 4

Используя исходный пример, вы можете затем вычислить диапазон как [4, ∞), не забывая использовать скобки и круглые скобки соответствующим образом.

Область и диапазон квадратичной функции

Общая форма квадратичной функции:

y = ax 2 + bx + c

Область — это все действительные значения x, для которых определена данная квадратичная функция.

Диапазон — это все действительные значения y для данного домена (реальные значения x).

Область квадратичной функции

Общая форма квадратичной функции:

y = ax 2 + bx + c

Областью определения любой квадратичной функции в приведенной выше форме являются все действительные значения.

Потому что в приведенной выше квадратичной функции y определено для всех действительных значений x.

Следовательно, область определения квадратичной функции в форме y = ax 2 + bx + c — это все действительные значения.

То есть

Домен = {x | x ∈ R}

Диапазон квадратичной функции

Чтобы узнать диапазон квадратичной функции в форме

y = ax 2 + bx + c,

, мы должны знать следующие два момента.

Это,

(i) Парабола открыта вверх или вниз

(ii) координата Y в вершине Параболы.

Давайте посмотрим, как узнать, открыт ли график (парабола) квадратичной функции вверх или вниз.

(i) Парабола открыта вверх или вниз:

y = ax 2 + bx + c

Если старший коэффициент или знак «a» положительный, парабола открыта вверх и «а» отрицательно, парабола открыта вниз.

(ii) Координата y в вершине:

Чтобы узнать координату y вершины, сначала мы должны найти значение «x», используя формулу, приведенную ниже.

x = -b / 2a

Теперь мы должны подставить x = -b / 2a в данную квадратичную функцию.

Итак, y — координата квадратичной функции

y = f (-b / 2a)

Как найти диапазон из двух вышеупомянутых вещей:

(i) Если парабола открыта вверх , диапазон — это все действительные значения, большие или равные

y = f (-b / 2a)

(i) Если парабола открыта вниз, диапазон — это все действительные значения, меньшие или равные

у = f (-b / 2a)

Практические задачи

Задача 1:

Найдите область определения и диапазон квадратичной функции, приведенной ниже.

y = x 2 + 5x + 6

Решение:

Область:

В квадратичной функции y = x 2 + 5x + 6, мы можем подставить любое действительное значение для x .

Потому что y определяется для всех реальных значений x.

Следовательно, область определения данной квадратичной функции — это все действительные значения.

То есть

Домен = {x | x ∈ R}

Диапазон:

Сравнивая данную квадратичную функцию y = x 2 + 5x + 6 с

y = ax 2 + bx + c

, получаем

a = 1

b = 5

c = 6

Поскольку старший коэффициент «a» положительный, парабола открыта вверх.

Найдите координату x вершины vertecx.

x = -b / 2a

Заменить 1 на a и 5 на b.

x = -5/2 (1)

x = -5/2

x = -2,5

Подставьте -2,5 вместо x в заданную квадратичную функцию, чтобы найти координату y в вершине.

y = (-2,5) 2 + 5 (-2,5) + 6

y = 6,25 — 12,5 + 6

y = — 0,25

Итак, координата y вершины равна -0,25

Поскольку парабола открыта вверх, диапазон — это все действительные значения, большие или равные -0.25

Диапазон = {y | y ≥ -0,25}

Чтобы лучше понять область определения и диапазон квадратичной функции, давайте посмотрим на график квадратичной функции y = x 2 + 5x + 6.

Когда мы посмотрим на график , ясно, что x (домен) может принимать любое реальное значение, а y (диапазон) может принимать все действительные значения, большие или равные -0,25

Задача 2:

Найдите область определения и диапазон квадратичной функции, приведенной ниже.

y = -2x 2 + 5x — 7

Решение:

Область:

В квадратичной функции y = -2x 2 + 5x — 7, мы можем подставить любое вещественное число значение для x.

Потому что y определяется для всех действительных значений x

Следовательно, область определения данной квадратичной функции — это все действительные значения.

То есть

Домен = {x | x ∈ R}

Диапазон:

Сравнивая данную квадратичную функцию y = -2x 2 + 5x — 7 с

y = ax 2 + bx + c

, получаем

a = -2

b = 5

c = -7

Поскольку старший коэффициент «a» отрицательный, парабола открыта вниз.

x = -b / 2a

Заменить -2 вместо a и 5 вместо b.

x = -5/2 (-2)

x = -5 / (- 4)

x = 5/4

x = 1,25

Замените 1,25 вместо x в данной квадратичной функции, чтобы найти y- координата в вершине.

y = -2 (1,25) 2 + 5 (1,25) — 7

y = -3,125 + 6,25 — 7

y = -3,875

Итак, координата y вершины равна -3,875.

Поскольку парабола открыта вниз, диапазон — это все действительные значения, большие или равные -3.875.

Диапазон = {y | y -3,875}

Чтобы лучше понять область определения и диапазон квадратичной функции, давайте посмотрим на график квадратичной функции y = -2x 2 + 5x — 7.

Когда мы смотрим на график, видно, что x (домен) может принимать любое реальное значение, а y (диапазон) может принимать все реальные значения, меньшие или равные -3,875

Помимо того, что описано в этом разделе, если вам нужны другие математические данные, воспользуйтесь нашим пользовательским поиском Google здесь.

Если у вас есть отзывы о наших математических материалах, напишите нам:

[email protected]

Мы всегда ценим ваши отзывы.

Вы также можете посетить следующие веб-страницы, посвященные различным вопросам математики.

ЗАДАЧИ СО СЛОВАМИ

Задачи со словами HCF и LCM

Задачи со словами для простых уравнений

Задачи со словами на линейных уравнениях

Задачи со словами на квадратных уравнениях

Алгебраные задачи со словами

Проблемы со словами в поездах

Проблемы со словами по площади и периметру

Проблемы со словами по прямой и обратной вариациям

Проблемы со словами по цене за единицу

Проблемы со словами по цене за единицу

Word задачи по сравнению ставок

Преобразование общепринятых единиц Word задачи

Преобразование метрических единиц Word задачи

Word задачи по простому проценту

Word задачи по сложным процентам

Word задачи по типам ngles

Дополнительные и дополнительные угловые проблемы со словами

Двойные проблемы со словами

Тригонометрические проблемы со словами

Процентные проблемы со словами

Проблемы со словами о прибылях и убытках

Разметка и разметка Задачи

Задачи с десятичными словами

Задачи со словами о дробях

Задачи со словами о смешанных фракциях

Одношаговые задачи с уравнениями со словами

Проблемы с линейными неравенствами

Задачи со словами

Проблемы со временем и рабочими словами

Задачи со словами на множествах и диаграммах Венна

Задачи со словами на возрастах

Проблемы со словами по теореме Пифагора

Процент числового слова pr проблемы

Word проблемы на постоянной скорости

Word проблемы на средней скорости

Word задачи на сумму углов треугольника 180 градусов

ДРУГИЕ ТЕМЫ

Сокращения прибылей и убытков

Сокращение в процентах

Сокращение в таблице времен

Сокращение времени, скорости и расстояния

Сокращение соотношения и пропорции

Домен и диапазон рациональных функций

Домен и диапазон рациональных функций функции с отверстиями

Графики рациональных функций

Графики рациональных функций с отверстиями

Преобразование повторяющихся десятичных дробей в дроби

Десятичное представление рациональных чисел

Поиск квадратного корня с помощью long di зрение

L.Метод CM для решения временных и рабочих задач

Преобразование задач со словами в алгебраические выражения

Остаток при делении 2 в степени 256 на 17

Остаток при делении 17 в степени 23 на 16

Сумма всех трехзначных чисел, делимых на 6

Сумма всех трехзначных чисел, делимых на 7

Сумма всех трехзначных чисел, делимых на 8

Сумма всех трехзначных чисел, образованных с использованием 1, 3 , 4

Сумма всех трех четырехзначных чисел, образованных ненулевыми цифрами

Сумма всех трех четырехзначных чисел, образованных с использованием 0, 1, 2, 3

Сумма всех трех четырехзначных чисел числа, образованные с использованием 1, 2, 5, 6

Определите домен и диапазон по графику

Результаты обучения

  • Найдите область и диапазон по графику и уравнению.
  • Укажите область и диапазон функций инструментария.

Другой способ определить область и диапазон функций — использовать графики. Поскольку домен относится к набору возможных входных значений, домен графа состоит из всех входных значений, показанных на оси [latex] x [/ latex]. Диапазон — это набор возможных выходных значений, которые отображаются на оси [latex] y [/ latex]. Имейте в виду, что если график выходит за пределы видимой части графика, домен и диапазон могут быть больше, чем видимые значения.

Мы можем заметить, что граф простирается по горизонтали от [latex] -5 [/ latex] вправо без границ, поэтому доменом является [latex] \ left [-5, \ infty \ right) [/ latex]. График по вертикали — это все значения диапазона [latex] 5 [/ latex] и ниже, поэтому диапазон равен [latex] \ left (\ mathrm {- \ infty}, 5 \ right] [/ latex]. Обратите внимание, что домен и диапазон всегда записываются от меньших к большим значениям или слева направо для домена и от нижней части графика до верхней части графика для диапазона.

Пример: поиск домена и диапазона из графика

Найдите домен и диапазон функции [латекс] f [/ латекс].

Показать решение

Мы можем заметить, что горизонтальная протяженность графа составляет от –3 к 1, поэтому домен [latex] f [/ latex] равен [latex] \ left (-3,1 \ right] [/ latex].

График по вертикали составляет от 0 до [latex] –4 [/ latex], поэтому диапазон составляет [latex] \ left [-4,0 \ right] [/ latex].

Пример: поиск области и диапазона по графику добычи нефти

Найдите домен и диапазон функции [латекс] f [/ латекс].

Показать решение

Входное количество по горизонтальной оси — «годы», которые мы представляем переменной [latex] t [/ latex] для времени. Выходное количество составляет «тысячи баррелей нефти в день», что мы представляем переменной [латекс] b [/ латекс] для баррелей. График может продолжаться влево и вправо за пределы того, что просматривается, но на основе видимой части графика мы можем определить домен как [латекс] 1973 \ le t \ le 2008 [/ latex], а диапазон — как примерно [латекс] 180 \ ле б \ ле 2010 [/ латекс].

В интервальной записи доменом является [латекс] [1973, 2008] [/ латекс], а диапазон — примерно [латекс] [180, 2010] [/ латекс]. Для области и диапазона мы аппроксимируем наименьшие и наибольшие значения, поскольку они не попадают точно на линии сетки.

Попробуй

По графику определите домен и диапазон, используя обозначение интервалов.

Показать решение

Домен = [латекс] [1950, 2002] [/ латекс] Диапазон = [латекс] [47 000 000, 89 000 000] [/ латекс]

Вопросы и ответы

Могут ли домен и диапазон функции совпадать?

Да.Например, домен и диапазон функции корня куба являются набором всех действительных чисел.

Область применения и набор функций инструментария

Теперь мы вернемся к нашему набору функций инструментария, чтобы определить домен и диапазон каждой из них.

Для постоянной функции [latex] f \ left (x \ right) = c [/ latex], домен состоит из всех действительных чисел; ограничений на ввод нет. Единственное выходное значение — это константа [latex] c [/ latex], поэтому диапазон — это набор [latex] \ left \ {c \ right \} [/ latex], который содержит этот единственный элемент.В обозначении интервалов это записывается как [latex] \ left [c, c \ right] [/ latex], интервал, который начинается и заканчивается [latex] c [/ latex].

Для функции идентификации [latex] f \ left (x \ right) = x [/ latex] нет ограничений на [latex] x [/ latex]. И домен, и диапазон представляют собой набор всех действительных чисел.

Для функции абсолютного значения [latex] f \ left (x \ right) = | x | [/ latex] нет ограничений на [latex] x [/ latex]. Однако, поскольку абсолютное значение определяется как расстояние от 0, выходные данные могут быть только больше или равны 0.{3} [/ latex], домен — это все действительные числа, потому что горизонтальная протяженность графика — это целая линия действительных чисел. То же самое относится к вертикальному экстенту графика, поэтому домен и диапазон включают все действительные числа.

Для обратной функции [latex] f \ left (x \ right) = \ frac {1} {x} [/ latex], мы не можем делить на 0, поэтому мы должны исключить 0 из домена. Кроме того, 1, деленная на любое значение, никогда не может быть 0, поэтому диапазон также не будет включать 0. В нотации конструктора множеств мы также могли бы написать [latex] \ left \ {x | \ text {} x \ ne 0 \ right \} [/ latex], множество всех действительных чисел, которые не равны нулю.{2}} [/ latex], мы не можем разделить на [latex] 0 [/ latex], поэтому мы должны исключить [latex] 0 [/ latex] из домена. Также нет [latex] x [/ latex], который может дать на выходе 0, поэтому 0 также исключается из диапазона. Обратите внимание, что результат этой функции всегда положительный из-за квадрата в знаменателе, поэтому диапазон включает только положительные числа.

Для функции квадратного корня [latex] f \ left (x \ right) = \ sqrt [] {x} [/ latex], мы не можем извлечь квадратный корень из отрицательного действительного числа, поэтому домен должен быть равен 0 или выше.Диапазон также исключает отрицательные числа, поскольку квадратный корень положительного числа [латекс] x [/ latex] определен как положительный, хотя квадрат отрицательного числа [латекс] — \ sqrt {x} [/ latex] также дает нам [латекс] х [/ латекс].

Для функции корня куба [latex] f \ left (x \ right) = \ sqrt [3] {x} [/ latex], домен и диапазон включают все действительные числа. Обратите внимание, что нет проблем с получением кубического корня или любого нечетно-целочисленного корня отрицательного числа, и результирующий результат будет отрицательным (это нечетная функция).

Внесите свой вклад!

У вас была идея улучшить этот контент? Нам очень понравится ваш вклад.

Улучшить эту страницуПодробнее

Как найти диапазон функции

Изучение того, как найти диапазон функции, может оказаться очень важным в алгебре и исчислении, потому что это дает вам возможность оценить, какие значения достигаются функцией.Другими словами, он позволяет найти набор всех изображений с помощью функции

Задача поиска точек, которые могут быть достигнуты функцией, очень полезна. Например, у вас может быть производственная функция \ (q (x) \), которая дает вам количество продукции, полученной для \ (x \) единиц ввода.

Мы хотели бы знать, сколько единиц ввода необходимо для производства \ (b \) единиц продукции.Другими словами, нам нужно найти \ (x \) так, чтобы \ (q (x) = b \), что является еще одним способом спросить, находится ли \ (b \) в диапазоне функции \ ( д (х) \).

В этом уроке мы больше сконцентрируемся на механике нахождения диапазона. Для более концептуального подхода к домену и диапазону вы можете

проверьте этот учебник

.


Алгебраический способ определения диапазона функции

Так же, как когда мы узнали, как вычислить область, не существует одного рецепта для нахождения диапазона, это действительно зависит от структуры функции \ (f (x) \).

Тем не менее, всегда будет использоваться один алгебраический прием.ТАКИМ способом вы найдете диапазон. Обращать внимание:

Скажем, нам нужно получить диапазон заданной функции \ (f (x) \). Затем мы рассмотрим типичное действительное число \ (y \) и попытаемся решить для \ (x \) следующее уравнение:

\ [f (x) = y \]

Нам нужно определить, для каких значений \ (y \) указанное выше уравнение может быть решено относительно \ (x \).Вот и все. Конечно, это может быть сложно сделать, в зависимости от структуры функции \ (f (x) \), но это то, что вам нужно сделать.

Итак, это алгебраический способ, способ найти диапазон функции без построения графиков.


ПРИМЕР 1

Найдите диапазон функции \ (\ displaystyle f (x) = \ frac {x + 1} {x-3} \):

ОТВЕЧАТЬ:

Мы действуем алгебраическим путем: пусть \ (y \) будет числом, и мы решим относительно \ (x \) в следующем уравнении: \ (f (x) = y \).Значение \ (y \) находится в диапазоне, если \ (f (x) = y \) может быть решено для \ (x \).

В этом случае мы имеем:

\ [\ большой f (x) = y \ Leftrightarrow \ frac {x + 1} {x-3} = y \]
\ [\ Rightarrow \, \, \, x + 1 = y \ left (x-3 \ right) \]
\ [\ Rightarrow \, \, \, x + 1 = yx-3y \]
\ [\ Rightarrow \, \, \, x-yx = -1-3y \]
\ [\ Rightarrow \, \, \, x \ left (1-y \ right) = — 1-3y \]
\ [\ Rightarrow \, \, \, x = \ frac {3y + 1} {y-1} \]

Следовательно, когда будет корректно определено \ (x \)? Почти для всех \ (y \), за исключением случая, когда \ (y = 1 \), потому что в этом случае у нас есть деление на \ (0 \).Следовательно, диапазон \ (f \) в этом случае — это вся вещественная линия, кроме 1.

Если мы используем обозначение интервалов, мы можем написать \ (Range (f) = (- \ infty, 1) \ cup (1, + \ infty) \). 2 — 4x + 3 \):

ОТВЕЧАТЬ:

Опять же, мы продолжаем использовать алгебраический способ, чтобы вы знали упражнение: Пусть \ (y \) будет числом, и мы решим для \ (x \) в следующем уравнении: \ (f (x) = y \).2–4 (1) (3-y)}} {2 (1)} \]
\ [\ Rightarrow \ displaystyle \, \, \, x = \ frac {4 \ pm \ sqrt {16-4 (3-y)}} {2} \]
\ [\ Rightarrow \ displaystyle \, \, \, x = \ frac {4 \ pm \ sqrt {16–12 + 4y}} {2} \]
\ [\ Rightarrow \ displaystyle \, \, \, x = \ frac {4 \ pm \ sqrt {4 + 4y}} {2} \]
\ [\ Rightarrow \ displaystyle \, \, \, x = 2 \ pm \ sqrt {1 + y} \]

Теперь, видя это последнее выражение, когда будет правильно определено \ (x \)? Нам нужно, чтобы аргумент квадратного корня был неотрицательным, поэтому нам нужно:

\ [1 + y \ ge 0 \]

что означает, что \ (y \ ge -1 \). 2 — 4 (2) + 3 = -1 \]

Поскольку минимальное значение, достигаемое параболой, равно \ (- 1 \), мы заключаем, что диапазон равен \ ([- 1, + \ infty) \), что совпадает с выводом, найденным алгебраически.2 — 4x + 3 \) делает его еще более ясным:

Мы можем видеть, что на основании графика минимум достигается в точке \ (x = 2 \), что в точности соответствует координате x вершины.

Риск использования графика для определения диапазона заключается в том, что вы потенциально можете неправильно прочитать критические точки на графике и дать неточную оценку того, где функция достигает своего максимума или минимума.


Другие стратегии поиска диапазона функции

Как мы видели в предыдущем примере, иногда мы можем найти диапазон функции, просто взглянув на ее график.

Например, вы хотите найти диапазон функции \ (f (x) = x + 3 \).График показан ниже:

На графике выше не показаны минимальные или максимальные точки. Более того, когда \ (x \) большое и положительное значение, значение функции также велико и положительно. И аналогично, когда \ (x \) очень отрицательно, значение функции также очень отрицательное.

Интуиция подсказывает, что функция может принимать любые отрицательные и положительные значения, выбирая достаточно большие (положительные или отрицательные) значения \ (x \).И тогда вывод состоит в том, что диапазон — это вся вещественная линия, которая равна \ ((- \ infty, + \ infty) \) с использованием обозначения интервалов.

Такой анализ верен с точки зрения результата, но ненадежен с точки зрения рассуждений. У «графического метода» поиска диапазона есть та же проблема: он привлекателен с интуитивной точки зрения, но довольно скуден с точки зрения содержания.

Обычно, если возможно, мы должны предпочесть аналитический / алгебраический путь.В этом примере нам нужно найти \ (x \):

\ [х + 3 = у \]

\ [\ Rightarrow \, \, x = y — 3 \]

Итак, есть ли какое-либо ограничение на \ (y \) для корректного определения \ (x \)? Вовсе нет, поэтому нет никаких ограничений на \ (y \), и можно сделать вывод, что диапазон — это вся вещественная линия.

Вы можете проверить эту статью, если хотите

знать, как найти домен функции

вместо.

Есть много веских алгебраических причин для нахождения диапазона, одна из них заключается в том, что он является частью процессов для

найти обратную функцию

.

Область, диапазон и квадратичные неравенства — The Math Doctors

(Новый вопрос недели)

В августе прошлого года мы долго обсуждали область определения и диапазон функций, включающих либо квадратичные функции, либо ограниченные области (или и то, и другое).Были задействованы два доктора математики, которые предложили разные способы решения одной и той же проблемы. Я отредактировал обсуждение, чтобы избежать смешения тем.

Квадратичное неравенство

Вот вопрос, от Mooija:

Привет, я пробовал это, но не могу найти правильный ответ.

Я просмотрел несколько видео, чтобы попытаться лучше понять, но это не помогло.

Ответ на вопрос: x ≥ 4 и x ≤ 3.

Пожалуйста, помогите мне разобраться, что я делаю не так.

Мне нужно найти максимально возможную область действия функции.

Найдите время, чтобы выявить любые ошибки, которые вы видите здесь.

Доктор Фентон ответил:

Вы смотрите нужную информацию, но неправильно ее анализируете. Вы правильно разложили квадратичный коэффициент и смотрите на знак каждого фактора, находя интервал, в котором каждый фактор неотрицателен.

Что вы упускаете из виду, так это то, что знак квадратичной определяется не знаком единственного множителя, а знаками всех множителей .

Один из способов решить проблему — отметить, что множитель (x — 3) положителен, когда x> 3, и множитель (x — 4) положителен, когда x> 4. Вы можете использовать эти две точки для отсечения числовая строка на три интервала : x <3; 3 <х <4; и x> 4. Затем посмотрите на знак квадратичной функции в каждом интервале.

 х-4 <0 х-4 <0 х-4> 0
   х-3 <0 х-3> 0 х-3> 0
-------------- + ------------------ + -------------
   3 4 

Произведение двух чисел положительно, если оба числа имеют одинаковый знак, и отрицательно, если числа имеют противоположные знаки.

Используя этот факт, можете ли вы теперь определить интервалы, в которых (x — 3) (x — 4) положительно? Поскольку вам нужны интервалы, в которых произведение неотрицательно, вы должны включить в свой ответ конечные точки каждого интервала, где квадратичная величина положительна.

Другой способ увидеть проблему — геометрический или графический. График y = (x — 3) (x — 4) представляет собой параболу, U-образную кривую. Вы ищите интервалы, в которых кривая находится на оси x или выше нее.Факторизация показывает, что кривая пересекает ось x при x = 3 и x = 4. Будучи U-образной (и открывающейся вверх), она спускается в левом интервале, пересекает в точке x = 3 и идет ниже оси x и остается внизу до x = 4 и, наконец, поднимается вправо от x = 4.

Таким образом, решением будет x ≥ 4 или x ≤ 3. (Муиджа сказал: « x ≥ 4 и x ≤ 3 ″, предположительно имея в виду объединение этих двух наборов. мы увидим ниже, похоже, это то, чему учили, хотя я считаю это неправильным.)

Конкретная ошибка Моуджи заключалась в делении на множитель; мы увидим это снова ниже.

Информацию об аналогичном исправлении подобной ошибки см .:

 Обращение нечетного неравенства 

Моиджа задал еще один вопрос:

Спасибо, что очень помогли. Я получаю это сейчас

У меня возникла еще одна проблема, которая по сути является продолжением этой, но с другим вопросом.

Не могли бы вы мне тоже помочь?

Вы видите, что вы мне объяснили, это домен ; теперь мне нужно найти диапазон с заданным доменом , и я не знаю, как это сделать.

Доктор Фентон ответил:

Я упоминал, что график этой квадратичной кривой представляет собой параболу, которая имеет U-образную форму, поэтому на графике будет самая низкая точка. Графически диапазон функции — это набор всех точек на оси Y, которые могут быть достигнуты, если идти горизонтально от точки на графике. Как только вы узнаете точку (x 0 , y 0 ) на графике, это число y 0 будет принадлежать диапазону, и любое значение y больше, чем y 0 , также будет в диапазоне.2 — \ frac {1} {4} \), равно \ (x \ ge — \ frac {1} {4} \). Оказывается, в этом нет необходимости…

Контекст: область радикальной функции

Доктор Рик вскочил, увидев недостающий аспект проблемы:

Привет, Муиджа, я хотел бы добавить кое-что к тому, что сказал доктор Фентон, потому что я не уверен, что мы действительно ответили на вопрос. Хотелось бы узнать, что именно задан вопросом!

Вы дали неравенство ,

x 2 — 7x + 12 ≥ 0

Вы показали «ответ на вопрос» как «x ≥ 4 и x ≤ 3», что говорит мне, что «вопрос» заключается в разрешении неравенства.Но затем вы добавили: «Мне также нужно найти максимально возможный домен функции ». Вы можете сказать нам, о какой функции идет речь?

Если это функция

f (x) = x 2 — 7x + 12

, тогда максимально возможный домен из f — это не то, что вы обсуждали с доктором Фентоном, а скорее набор всех действительных чисел. Этот вопрос не имеет ничего общего с положительным или отрицательным значением f (x), а только с тем, можно ли вычислить f (x).

Теперь, если проблема связана с функцией

г (х) = √ f (х)

= √ (x 2 — 7x + 12)

, то вы правильно сделали, определив интервал (интервалы), на котором f (x) ≥ 0. Я просто не могу быть уверен, что вы делаете именно это, потому что вы нам не сказали. Вот почему я надеюсь, что вы проясните, о чем на самом деле говорит проблема.

Mooija ответил:

Хорошо, чтобы уточнить, я попросил помощи с вопросом 7g.Это прямо сейчас из моего учебника:

Решения:

Доктор Рик угадал! Он ответил:

Спасибо, Муиджа, это ответ на мой вопрос! Вы нашли максимально возможную область определения функции

f (x) = √ (x 2 — 7x + 12)

, значит, вы поступили правильно, решив неравенство, утверждая, что величина под радикалом неотрицательна.

Я вижу, что вам не предлагается найти диапазон приведенной выше функции, что неинтересно — это набор всех неотрицательных чисел.Нет необходимости находить минимум квадратичной функции, поскольку этот минимум не встречается в области определения f. Если это непонятно, мы можем обсудить любые ваши вопросы по этому поводу.

Если у вас есть проблемы того типа, который вы описали (поиск диапазона функции с ограниченным доменом), не стесняйтесь показать нам такую ​​проблему с вашей работой, и мы сможем ее обсудить. Процесс определения диапазона может сильно зависеть от деталей конкретной функции .Однако все части проблемы 8, которые я вижу, являются линейными функциями, поэтому, если это то, о чем вы спрашиваете, это будет относительно легко. Выберите одну проблему и попробуйте, и я или доктор Фентон решим ее оттуда.

Другое квадратное неравенство

Mooija позже показал работу над проблемой 7e; Я вставлю это обсуждение сюда, а после этого мы перейдем к 8.

Мне очень жаль, но я все еще не понимаю, как решить домен функции.

В аттаче пробовал два разных метода; первый был неправ.

Во втором методе я считаю а) правильным, но в книге сказано, что б) правильно, что означает, что я полностью ошибаюсь, поэтому, пожалуйста, скажите мне, что я делаю не так.

Ошибка здесь такая же, как и раньше. Доктор Фентон ответил с другой точки зрения:

Вы, , разделили неравенство x (x — 4) ≥ 0 на x в левой половине второй строки, получив x ≥ 0 в левой части третьей строки, а затем разделили x (x — 4) ≥ 0 на (x — 4) в правой половине второй строки, получая x ≥ 0 в правой части третьей строки.

Этот метод упрощения неравенства не работает так же, как для уравнений. Проблема в том, что , если вы разделите неравенство на отрицательное число, полученное неравенство не будет истинным . Например, 4 <6 является истинным неравенством, и если мы разделим неравенство на положительное число, 2, получится неравенство 2 <3, что также верно. Но если мы разделим 4 <6 на отрицательное число -2, получится неравенство -2 <-3, что неверно. Когда вы делите x (x - 4) ≥ 0 на x, результирующее неравенство x - 4 ≥ 0 истинно, только если x> 0.

В то время как этот процесс можно заставить работать, рассматривая случаи , где x> 0 и где x <0, он становится беспорядочным. Более простой подход - рассмотреть каждый фактор в задаче отдельно, , и найти значения x, которые делают каждое неравенство истинным: в этом случае вы найдете, что x ≥ 0, а где x — 4 ≥ 0. Первое неравенство имеет вид уже решено: x ≥ 0; а второй становится x ≥ 4.

Мы можем изобразить эти решения на числовой прямой:

 х ≥ 0
               • ------------------------------------------------- >
  х-4 ≥ 0
                                   • ----------------------------->
+ ---- + ---- + ---- + ---- + ---- + ---- + ---- + ---- + ---- + ---- + ---- + ---- + ---- +

... -2-1 0 1 2 3 4 5 6 7 8 ... 

Верхняя строка показывает решение x ≥ 0 в том смысле, что все точки на числовой прямой под верхней линией являются решениями x ≥ 0. Точно так же все точки под нижней строкой являются решениями x ≥ 4 (или х — 4 ≥ 0). Произведение x (x — 4) ≥ 0 будет истинным, если оба множителя имеют одинаковый знак: оба положительны или оба отрицательны (или если один множитель равен 0). Точки на числовой прямой, где это верно, — это точки под обеими линиями или не под любой из них.Точки под обеими линиями — это точки, где x ≥ 4 (мы включаем 4, потому что множитель x — 4 = 0. Точки, не находящиеся ни под одной из линий, — это точки, где x <0, и мы включаем x = 0, потому что мы только требуя, чтобы x (x - 4) = 0, что верно при x = 0. (Если бы неравенство было x (x - 4)> 0, то мы не включали бы ни x = 0, ни x = 4.)

Имеет ли это смысл для вас?

Теперь мы можем перейти к вопросу о диапазоне.

Диапазон с ограниченной областью (линейный)

Моиджа сказал о проблеме 8 и диапазоне:

По правде говоря, я застрял на цифре 8, потому что я не знаю, как найти диапазон с областью всех реальных значений .

Если домен состоит из реальных значений, это означает, что домен в основном бесконечен, как и диапазон.

Mooija неверно сформулировал задачу 8, которая определяла домен как «все положительных вещественных чисел». Это должно было вызвать недоразумения.

Доктор Рик рассмотрел задачу 8 (a):

Вы хотите понять, как найти диапазон функции, когда домен ограничен — когда он на меньше, чем все действительные числа . В задаче 8 домен ограничен всеми положительными действительными числами; Таким образом, найденный вами ответ нам не помогает — он говорит о другом типе проблемы.

Теперь давайте посмотрим на одну из частей проблемы 8. Если мы определим

f (x) = 2x + 7 с доменом {все положительные числа}

, то мы спрашиваем, если x> 0, какое неравенство применяется к 2x + 7? Посмотрите, как я начинаю с x> 0, чтобы получить желаемый результат:

х> 0

2x> 2 (0) (обе части умножены на 2, что положительно)

2x> 0 (поскольку 2 (0) = 0)

2x + 7> 0 + 7 (прибавлено 7 с обеих сторон)

2x + 7> 7

Таким образом, я обнаружил, что 2x + 7, то есть f (x), должно быть больше 7.Это диапазон f. Все, что я сделал, — это пошагово «построил» функцию f (x) = 2x + 7, начиная с x, и работал с неравенством, как я пошел, чтобы получить неравенство, эквивалентное x> 0.

Есть и другие способы подойти к проблеме поиска области определения ограниченной линейной функции. Например, мы можем мыслить графически — это поможет вам лучше понять, что происходит. Но, по крайней мере, это отправная точка для обсуждения.

Подход «наращивания» здесь хорошо работает, но мы увидим, как Муиджа пытается наивно использовать его в квадратичном случае ниже.

Диапазон с ограниченной областью (квадратичный)

Моиджа показал нам остальную часть задачи 8, которая касается квадратичных функций (и, следовательно, возвращает нас к исходному вопросу о диапазоне):

К номеру 8 относятся и другие части.

Мне удалось решить их все, кроме одного, последнего из числа 8.

Доктор Фентон попросил показать работу Моиджи, подозревая, что метод может быть неправильным для всех трех, даже если ответы были правильными:

Что касается проблемы 8 (f), вы говорите, что у вас не было проблем с предыдущими частями проблемы 8.Однако последние три части задачи 8 очень похожи. Можете ли вы показать мне, как вы нашли диапазон в задаче 8 (d) или 8 (e)?

Моиджа ответил, показывая неправильную попытку подхода «наращивания», который, как оказалось, сработал для (d) и (e), но не был достаточно надежным для (f):

Я выполнил (d) и (e), но не смог получить (f) правильно, хотя я использовал тот же метод, что и в (e).

Доктор Фентон некоторое время занимался этим, забыв, что область в задаче 8 ограничена положительными числами, так что просто найти вершину недостаточно.Доктор Рик предложил два способа справиться с этим:

Привет, Муиджа, это снова доктор Рик. Это, наверное, сбивает с толку из-за всего этого, но я думаю, что нужен еще один глаз. Позвольте мне еще раз взглянуть на проблему 8 (e).

Вам нужно найти диапазон функции

f (x) = (x + 2) 2 — 1; домен: x> 0

Вот что я бы сделал — аналогично тому, что я сделал с линейной функцией ранее. Я начинаю с неравенства, определяющего диапазон, и меняю его шаг за шагом, делая правильные вещи (вещи, которые производят эквивалентные неравенства):

[1] x> 0

[2] x + 2> 0 + 2 = 2

[3] (x + 2) 2 > 2 2 = 4

[4] (x + 2) 2 — 1> 4 — 1 = 3

Таким образом, я показал, что f (x)> 3 для всех x в области x> 0.Это ответ, который дала книга.

Мы должны быть осторожны при работе с квадратиками — шаг возведения в квадрат выше, шаг [3], действителен только , потому что мы уже знаем, что возведенная в квадрат величина (x + 2) положительна . Если бы домен был x <0 вместо x> 0, тот же набор шагов (который, казалось бы, давал результат f (x) <3) был бы неправильным - правильный диапазон на самом деле был бы x> -1 (диапазон неограниченная квадратичная).

График функции может помочь вам увидеть, что происходит, как я и предлагал в отношении линейных функций.Красный график на прикрепленном изображении: f (x) = (x + 2) 2 — 1 с учетом ограничения домена x> 0; вы увидите, что наименьшее значение y на этой кривой равно y = 3. Пунктирная кривая — это оставшаяся часть квадратичной кривой, минимальное значение y которой равно y = -1.

Аналогичные, но разные

Сейчас Муиджа попробовала 8 (f):

Хорошо, я сделал 8 (f), используя оба ваших метода:

Я думаю, что с методом доктора Рика я сделал что-то не так.

С доктором.Фентон: Я пропустил несколько шагов.

Дело в том, что даже если эти ошибки будут исправлены, они все равно будут неправильными, потому что f использует неравенство ≥, а остальные нет, вы можете сказать мне, почему?

Упомянутый здесь метод доктора Фентона не учитывает ограниченную область, а просто использует вершину. Он объяснил:

Ваша ошибка в методе доктора Рика состоит в том, что вы возводили в квадрат (x — 1)> -1, чтобы получить (x — 1) 2 > 1. Если a 2 < b 2 , если оба значения a и b положительны .Например, -1 <0, но неверно, что (-1) 2 <0 2 : это означает, что 1 <0.

Вы заключаете, что (x — 1) 2 > 1 для всех x> 0, но это неверно. Если x = 1, то x — 1 = 0. поэтому (x — 1) 2 = 0 и (x — 1) 2 +2 ≥ 2. (Если вы знаете, что такое возрастающие функции, то вы лечите (x — 1) 2 +2, как если бы оно увеличивалось на интервале x> 0, но на самом деле оно уменьшается при 0 1.)

Когда вы пытаетесь использовать «мой» метод, вы переходите от x> 0 напрямую к (x — 1) 2 +2> 2. Это говорит о том, что вы думаете, что, поскольку x> 0, то (x — 1) 2 > 0, но когда x = 1, (x — 1) 2 = 0, поэтому (x — 1) 2 + 2 ≥ 2.

Это восходит к тому, что я сказал выше, что если a 2 2 , только если a и b оба положительны. В задаче 8 (e) вам нужен диапазон (x + 2) 2 — 1 для x> 0.Если x> 0, то x + 2> 2, поэтому вы можете заключить, что (x + 2) 2 > 4, поскольку и (x + 2), и 2 положительны. В задаче 8 (f) квадратичная функция равна (x — 1) 2 , поэтому, когда 0 -1. Но поскольку неверно, что и x — 1, и -1 положительны, вы не можете заключить, что (x — 1) 2 > (-1) 2 .

Затем Доктор Рик добавил:

Как вы помните, после того, как я показал свой метод для задачи 8 (e), я указал, что он не всегда работает — только тогда, когда известно, что обе величины, которые нужно возвести в квадрат, положительны .Как сказал д-р Фентон, проблема 8 (f) — одна из проблем, к которым применимо это предостережение. Так что вам понадобится другой метод…

Я не знаю, каким методам решения этих проблем вас учат. Лично я рисовал бы график параболы и принимал бы решения на основе этого. Если вы научились рисовать параболы, то знаете, что график функции

f (x) = (x — 1) 2 + 2

— парабола с вершиной в определенной точке, которая открывается вверх.Это говорит нам о том, что, независимо от ограничений области, значение f (x) не может быть меньше координаты y вершины. Если вершина находится справа от оси y (и, следовательно, в области определения функции), тогда f (x) действительно может иметь это минимальное значение. И в этом случае, поскольку вся правая половина параболы находится в области, нет верхнего предела для f (x).

Если у вас нет подготовки для выполнения работы, которую я только что изложил, я хотел бы увидеть некоторую информацию о том, что вы узнали, особенно о том, как подобные проблемы решались в вашем учебнике или классе.

Вот график этой функции:

Мы видим, что минимальное значение находится не при x = 0, как раньше, а в вершине параболы.

Эти проблемы могут быть довольно незаметными. Я бы рекомендовал хотя бы сверить ваш ответ с наброском графика, даже если вы будете осторожно использовать строго алгебраический метод!

Область и диапазон рациональных функций

В

домен

из

функция

ж

Икс

— это набор всех значений, для которых определена функция, а

диапазон

функции — это набор всех значений, которые

ж

берет.

Рациональная функция — это функция вида

ж

Икс

знак равно

п

Икс

q

Икс

, где

п

Икс

а также

q

Икс

являются многочленами и

q

Икс

0

.

Область определения рациональной функции состоит из всех действительных чисел

Икс

кроме тех, для которых знаменатель

0

. Чтобы найти эти

Икс

значения, которые необходимо исключить из области определения рациональной функции, приравнять знаменатель к нулю и решить для

Икс

.

Например, домен

родительская функция

ж

Икс

знак равно

1

Икс

это набор всех действительных чисел, кроме

Икс

знак равно

0

. Или область определения функции

ж

Икс

знак равно

1

Икс

4

это набор всех действительных чисел, кроме

Икс

знак равно

4

.

Теперь рассмотрим функцию

ж

Икс

знак равно

Икс

+

1

Икс

2

Икс

2

.По упрощению, когда

Икс

2

он становится линейной функцией

ж

Икс

знак равно

Икс

+

1

. Но исходная функция не определена в

Икс

знак равно

2

. Это оставляет график с дырой, когда

Икс

знак равно

2

.

Один из способов найти диапазон рациональной функции — найти область определения обратной функции.

Другой способ — нарисовать график и определить диапазон.

Снова рассмотрим родительскую функцию

ж

Икс

знак равно

1

Икс

. Мы знаем, что функция не определена, когда

Икс

знак равно

0

.

В виде

Икс

0

по обе стороны от нуля,

ж

Икс

. Аналогично, как

Икс

±

,

ж

Икс

0

.

График приближается

Икс

-ось как

Икс

стремится к положительной или отрицательной бесконечности, но никогда не касается

Икс

-ось.То есть функция может принимать все реальные значения, кроме

0

.

Итак, диапазон функции — это набор действительных чисел, кроме

0

.


Пример 1:

Найдите домен и диапазон функции

у

знак равно

1

Икс

+

3

5

.

Чтобы найти исключенное значение в области определения функции, приравняйте знаменатель к нулю и решите для

Икс

.

Икс

+

3

знак равно

0

Икс

знак равно

3

Итак, область определения функции — это набор действительных чисел, кроме

3

.

Диапазон функции такой же, как и область определения обратной функции. Итак, чтобы найти диапазон, определите обратную функцию.

Поменять местами

Икс

а также

у

.

Икс

знак равно

1

у

+

3

5

Решение для

у

ты получаешь,

Икс

+

5

знак равно

1

у

+

3

у

+

3

знак равно

1

Икс

+

5

у

знак равно

1

Икс

+

5

3

Итак, обратная функция

ж

1

Икс

знак равно

1

Икс

+

5

3

.

Исключенное значение в области определения обратной функции можно определить, приравняв знаменатель к нулю и решив для

Икс

.

Икс

+

5

знак равно

0

Икс

знак равно

5

Итак, область определения обратной функции — это набор действительных чисел, кроме

5

. То есть диапазон данной функции — это набор действительных чисел, кроме

5

.

Следовательно, область определения данной функции равна

{

Икс

|

Икс

3

}

и диапазон

{

у

|

у

5

}

.


Пример 2:

Найдите домен и диапазон функции

у

знак равно

Икс

2

3

Икс

4

Икс

+

1

.

Используйте графический калькулятор, чтобы построить график функции.

Когда вы множите числитель и отменяете ненулевые общие множители, функция сводится к линейной функции, как показано.

у

знак равно

Икс

+

1

Икс

4

Икс

+

1

знак равно

Икс

+

1

Икс

4

Икс

+

1

знак равно

Икс

4

Итак, график линейный с дырой в

Икс

знак равно

1

.

Используйте график, чтобы определить домен и диапазон.

Функция не определена для

Икс

знак равно

1

. Итак, домен

{

Икс

|

Икс

1

}

или же

,

1

1

,

.

Диапазон функции:

{

у

|

у

k

где

у

1

знак равно

k

}

.

Для

Икс

1

, функция упрощается до

у

знак равно

Икс

4

.Функция не определена в

Икс

знак равно

1

или функция не принимает значение

1

4

знак равно

5

. Это,

k

знак равно

5

.

Следовательно, диапазон функции равен

{

у

|

у

5

}

или же

,

5

5

,

.

Асимптоты рациональной функции:

An

асимптота

это линия, к которой график функции приближается, но никогда не касается. В родительской функции

ж

Икс

знак равно

1

Икс

, как

Икс

— а также

у

-оси — это асимптоты. График родительской функции будет приближаться к асимптотам, но никогда не касается их.

Чтобы найти вертикальную асимптоту рациональной функции, приравняйте знаменатель к нулю и решите относительно

Икс

.

Если степень многочлена в числителе меньше степени знаменателя, то горизонтальная асимптота — это

Икс

-ось или

у

знак равно

0

.

Функция

ж

Икс

знак равно

а

Икс

,

а

0

имеет тот же домен, диапазон и асимптоты, что и

ж

Икс

знак равно

1

Икс

.

Теперь график функции

ж

Икс

знак равно

а

Икс

б

+

c

,

а

0

гипербола, симметричная относительно точки

б

,

c

.Вертикальная асимптота функции равна

Икс

знак равно

б

а горизонтальная асимптота равна

у

знак равно

c

.

В более общем виде функция

ж

Икс

знак равно

а

Икс

+

б

c

Икс

+

d

имеет вертикальную асимптоту при

Икс

знак равно

d

c

и горизонтальная асимптота при

у

знак равно

а

c

.В более общем смысле, если и числитель, и знаменатель имеют одинаковую степень, то горизонтальная асимптота будет иметь вид

у

знак равно

k

где

k

— отношение старшего коэффициента числителя к знаменателю.

Если степень знаменателя на единицу меньше степени числителя, то функция имеет наклонную асимптоту.


Пример 3:

Найдите вертикальную и горизонтальную асимптоты функции

ж

Икс

знак равно

5

Икс

1

.

Чтобы найти вертикальную асимптоту, приравняйте знаменатель к нулю и решите относительно

Икс

.

Икс

1

знак равно

0

Икс

знак равно

1

Итак, вертикальная асимптота равна

Икс

знак равно

1

Поскольку степень многочлена в числителе меньше степени знаменателя, горизонтальная асимптота имеет вид

у

знак равно

0

.

Область и диапазон функции

Определения домена и диапазона

Домен

Домен из
функция — это полный набор возможных значений
независимой переменной.

На простом английском языке это определение означает:

Домен — это совокупность всех возможных
x — значения, которые сделают функцию
«работать», и будет выводить реальные y -значения.

При нахождении домена запомните:

  • Знаменатель (внизу) дроби не может быть
    ноль
  • Число под знаком квадратного корня должно быть
    положительный
    в этом разделе

Пример 1а

Вот график y = sqrt (x + 4):

12345-1-2-3-4123xy

Домен: `x> = — 4`

Область определения этой функции — `x ≥ −4`, поскольку x не может быть меньше, чем` −4`.Чтобы понять, почему, попробуйте использовать в калькуляторе некоторые числа меньше, чем «−4» (например, «−5» или «−10»), и некоторые числа, превышающие «−4» (например, «−2» или «8»). Единственные, которые «работают» и дают нам ответ, — это те, которые больше или равны «−4». Это сделает число под квадратным корнем положительным.

Примечания:

  1. Замкнутый (закрашенный) кружок в точке `(-4, 0)`. Это указывает на то, что домен «запускается» в этот момент.
  2. Мы видели, как рисовать подобные графики в разделе 4, График функции.2 = х — 2.

Как найти домен

В общем, мы определяем область каждой функции, ища те значения независимой переменной (обычно x ), которые разрешено использовать для . (Обычно нам нужно избегать 0 в нижней части дроби или отрицательных значений под знаком квадратного корня).

Диапазон

Серия из
функция — это полный набор всех возможных
результирующих значений зависимой переменной ( y, обычно ) после того, как мы подставили домен.

На простом английском языке это определение означает:

Диапазон — это результат
y- значений, которые мы получаем после подстановки всех возможных x -значений.

Как найти ассортимент

  • Диапазон функции — это разброс возможных y -значений (от минимального y -значения до максимального y -значения)
  • Подставьте различные значения x в выражение для y на
    посмотреть, что происходит.(Спросите себя: всегда ли и положительны? Всегда отрицательны? Или, может быть, не равны определенным значениям?)
  • Убедитесь, что вы ищете минимум и максимум значений y .
  • Нарисуйте эскиз ! С точки зрения математики, картина стоит тысячи слов.

Пример 1б

Вернемся к примеру выше, `y = sqrt (x + 4)`.

Мы замечаем, что кривая находится либо на горизонтальной оси, либо над ней.Независимо от того, какое значение x мы попробуем, мы всегда получим нулевое или положительное значение y . Мы говорим, что диапазон в этом случае равен y ≥ 0.

12345-1-2-3-4123xy

Диапазон: `y> = 0`

Кривая продолжается всегда вертикально, за пределы того, что показано на графике, поэтому диапазон — это все неотрицательные значения `y`.

Пример 2

График кривой y = sin x показывает, что диапазон находится между -1 и 1.

12345-1-2-3-4-5-6-71-1xy

Диапазон: `-1

Область y = sin x — это «все значения x », поскольку нет никаких ограничений на значения для x . (Введите любое число в функцию «sin» в вашем калькуляторе. Любое число должно работать и даст вам окончательный ответ от -1 до 1.)

Эксперимент с калькулятором и наблюдение кривой показывают, что диапазон составляет y между -1 и 1.Мы могли бы записать это как −1 ≤ y ≤ 1.

Откуда взялся этот график? Мы узнаем о графиках sin и cos позже в Графах греха x и cos x

Примечание 1: Поскольку мы предполагаем, что для значений x должны использоваться только действительные числа, числа, которые приводят к делению на ноль или к мнимым числам (которые возникают при нахождении квадратного корня из отрицательное число) не включаются.В главе «Комплексные числа» более подробно рассказывается о мнимых числах, но мы не включаем такие числа в эту главу.

Примечание 2: При выполнении примеров квадратного корня многие люди спрашивают: «Разве мы не получаем 2 ответа, один положительный и один отрицательный, когда мы находим квадратный корень?» Квадратный корень имеет не более одного значения, а не два. См. Это обсуждение: Квадратный корень 16 — сколько ответов?

Примечание 3: Мы говорим о домене и диапазоне функций , которые имеют не более одно y -значение для каждого x -значения, а не отношений (которые могут иметь более одного .).

Поиск домена и диапазона без использования графика

Всегда намного проще определить домен и диапазон, считывая его с графика (но мы должны убедиться, что мы увеличиваем и уменьшаем масштаб графика, чтобы убедиться, что мы видим все, что нам нужно видеть). 2-9),` без использования графика.2-9`, которое, как мы понимаем, можно записать как `(x + 3) (x-3)`. Таким образом, наши значения для `x` не могут включать` -3` (из первой скобки) или `3` (из второй).

В любом случае нам не нужно беспокоиться о `-3`, потому что на первом шаге мы решили, что` x> = -2`.

Таким образом, домен для этого случая — `x> = -2, x! = 3`, который мы можем записать как` [-2,3) uu (3, oo) `.

Для определения диапазона мы рассматриваем верхнюю и нижнюю части дроби отдельно.

Числитель: Если `x = -2`, верхняя часть имеет значение` sqrt (2 + 2) = sqrt (0) = 0`.2-9) `приближается к` 0`, поэтому `f (x)` переходит в `-oo`, когда приближается к` x = 3`.

Для `x> 3`, когда` x` просто больше, чем `3`, значение дна чуть больше` 0`, поэтому `f (x)` будет очень большим положительным числом.

Для очень большого `x` верхний край большой, но нижний будет намного больше, поэтому в целом значение функции будет очень маленьким.

Таким образом, мы можем заключить, что диапазон равен `(-oo, 0] uu (oo, 0)`.

Посмотрите на график (который мы все равно рисуем, чтобы убедиться, что мы на правильном пути):

Показать график

Мы можем видеть на следующем графике, что действительно домен равен «[-2,3) uu (3, oo)» (который включает «-2», но не «3»), а диапазон — «все значения из `f (x)`, кроме `F (x) = 0`.2-9) `.

Сводка

В общем, мы определяем домен по
ищем те значения независимой переменной (обычно x ), которые мы разрешили использовать . (Мы должны избегать 0 в нижней части дроби или отрицательных значений под знаком квадратного корня).

Диапазон находится путем нахождения результирующих значений y после замены возможных значений x .

Упражнение 1

Найдите домен и диапазон для каждого из следующих.2+ 2`.

Ответ

Домен: Функция

f ( x ) = x 2 + 2

определен
для всех реальных значений x (поскольку нет ограничений на значение x ).

Следовательно, область `f (x)` равна

«все реальные значения x «.

Диапазон: Поскольку x 2 никогда не бывает отрицательным,
x 2 + 2 никогда не меньше 2

Следовательно, диапазон `f (x)` равен

«все действительные числа` f (x) ≥ 2` «.

Мы видим, что x может принимать любое значение на графике, но результирующие значения y = f ( x ) больше или равны 2.

123-1-2-312345678910-1xf (x)

Диапазон: `y> = 2`

Домен: Все `x`

Примечание

  1. При построении графиков важно обозначить оси как . Это помогает понять, что представляет собой график.
  2. Мы видели, как рисовать такие графики в Графике функции.

(б) `f (t) = 1 / (t + 2)`

Ответ

Домен: Функция

`f (t) = 1 / (t + 2)`

не определено для т =
-2, так как это значение приведет к делению на ноль. (Внизу дроби будет 0.)

Следовательно, домен из f ( t ) равен

«все
вещественные числа кроме -2 «

Диапазон: Независимо от того, насколько большим или малым станет т ,
f ( t ) никогда не будет равно нулю.

[ Почему? Если мы попытаемся решить уравнение относительно 0, произойдет следующее:

`0 = 1 / (t + 2)`

Умножаем обе стороны на ( t + 2) и получаем

`0 = 1`

Это невозможно.]

Таким образом, диапазон из f ( t ) равен

«все
вещественные числа кроме нуля ».

На графике видно, что функция не определена для t = -2 и что функция (значения y ) принимает все значения, кроме 0.

1234-1-2-3-4-5-6-712345-1-2-3-4-5tf (t)

Домен: Все `t ≠ -2`

Диапазон: Все `f (t) ≠ 0`

(c) `g (s) = sqrt (3-s)`

Ответ

Функция

`г (т) = sqrt (3-с)`

не определен для реального
числа больше 3, что приведет к мнимым значениям
для г ( с ). 2 + 4` для
`x> 2`

Ответ

Функция `f (x)` имеет область из
«все действительные числа,` x> 2` «, как определено в вопросе.(Здесь не используются квадратные корни из отрицательных чисел или деления на ноль.)

Чтобы найти диапазон :

  • Когда `x = 2`,` f (2) = 8`
  • Когда x увеличивается с `2`,` f (x) `становится
    больше, чем `8` (попробуйте подставить некоторые числа, чтобы понять, почему)

Следовательно, диапазон — «все действительные числа,` f (x)> 8` «

Вот график функции с открытым кружком в точке «(2, 8)», указывающим, что домен не включает «x = 2», а диапазон не включает «f (2) = 8».

123456510152025xf (x) (2, 8)

Домен: Все `x> 2`

Диапазон:
Все `f (x)> 8`

Функция является частью параболы. [Подробнее о параболе.]

Упражнение 2

Мы запускаем шар в воздух и находим
высота h , в метрах, как функция времени
т , в секундах, равно

ч = 20 т — 4,9 т 2

Найдите домен и диапазон для функции
ч ( т ).

Ответ

Как правило, отрицательные значения времени не имеют
имея в виду. Кроме того, нам нужно предположить, что снаряд попадает в землю, а затем останавливается — он не уходит под землю.

Итак, нам нужно рассчитать, когда он упадет на землю. Это будет, когда h = 0. Итак, решаем:

20 т — 4,9 т 2 = 0

Факторинг дает:

(20 — 4.9 т ) т = 0

Это верно, когда

`t = 0 \» s «`,

или

`t = 20/4.9 = 4.082 текст (ы) `

Следовательно, область функции h равна

«все реально
значения t такие, что `0 ≤ t ≤ 4.082`»

Из выражения функции видно, что это парабола с вершиной вверх. (Это имеет смысл, если вы думаете о подбрасывании мяча вверх. Он поднимается на определенную высоту, а затем падает обратно.)

Какое максимальное значение ч ? Воспользуемся формулой максимума (или минимума) квадратичной функции.

Значение т. дает максимум

.

`t = -b / (2a) = -20 / (2 xx (-4.9)) = 2.041 с`

Таким образом, максимальное значение равно

.

20 (2,041) — 4,9 (2,041) 2 = 20,408 м

Наблюдая за функцией h , мы видим, что по мере увеличения t , h сначала увеличивается до максимума.
20,408 м, затем ч снова уменьшается до нуля, как и ожидалось.

Следовательно, диапазон h равен

«все реально
числа, `0 ≤ h ≤ 20,408`»

Вот график функции ч :

1234565101520-5-й (t)

Домен: `0

Диапазон:
`0

Функции, определяемые координатами

Иногда у нас нет непрерывных функций. Что нам делать в этом случае? Давайте посмотрим на пример.

Упражнение 3

Найдите область и диапазон функции, определенной координатами:

`{(−4, 1), (−2, 2.5), (2, −1), (3, 2)} `

Ответ

Область — это просто следующие значения x : `x = {−4, −2, 2, 3}`

Диапазон состоит из следующих значений `f (x)`: `f (x) = {−1, 1, 2, 2.5}`

Вот график нашей разрывной функции.

1234-1-2-3-41234-1-2-3-е (т) (3, 2) (2, -1) (- 4, 1)

(-2, 2,5)

.

Добавить комментарий

Ваш адрес email не будет опубликован. Обязательные поля помечены *